*NURSING > HESI > PREDICTOR HESSI 05/2020 With All the Answers (All)

PREDICTOR HESSI 05/2020 With All the Answers

Document Content and Description Below

PREDICTOR HESSI 05/2018 1. An older adult resident of a long-term care facility has a 5-year history of hypertension. The client has a headache and rate the pain 5 on a pain scale 0 to 10. The clie... nt‟s blood pressure is currently 142/89. Which interventions should the nurse implement? (Select all that apply) A. Administer a daily dose of lisinopril as scheduled. B. Assess the client for postural hypotension. C. Notify the healthcare provider immediately D. Provide a PRN dose of acetaminophen for headache E. Withhold the next scheduled daily dose of warfarin. 2. The nurse mixes 250 mg of D5W dobutamine and plans to administer the solution at a rate of 15 mcg/kg/minute to a client weighing 110 pounds. The nurse should set the infusion pump to administer how many ml per hour? (Enter numeric value only rounding is required, round to the nearest whole number) 3. Several days after having a computed tomography scan (CT scan) with contrast for lower abdominal pain, an older male adult is admitted with acute kidney injury (AKI). The client‟s urinary output is 10ml/hour, his heart rate is 125 beats/minute, and basilar crackles are audible. Which laboratory value in the client‟s electronic record requires intervention by the nurse? A. Spot collection for Urine sodium 14 mEq/L (14mmol/L) B. Serum creatinine 2.1 mg/dl (186 mcmol/L) C. Blood Urea Nitrogen (BUN) 19 mg/dl (6.8 mmol/L) D. Potassium 6.8 MEq/L (6.8 mmol/L) 4. A male client is admitted to the hospital with a medical diagnosis of peptic ulcers. The nurse should inform the client that he is most likely to experience the greatest pain and discomforting which situation? A. After consuming a high-fat meal B. When the stomach is full. C. After consuming an extremely cold beverage D. When the stomach is empty 5. 5. A client in the third trimester of pregnancy reports that she feels some "lumpy places" in her breasts and that her nipples sometimes leak a yellowish fluid. She has an appointment with her healthcare provider in two weeks. What action should the nurse take? A Reschedule the client's prenatal appointment for the following day B Recommend that the client start wearing a supportive brassiere C Tell the client to begin nipple stimulation to prepare for breast feeding D Explain that this normal secretion can be assessed at the next visit 6. The charge nurse is planning for the shift and has a registered nurse (RN) and a practical nurse (PN) on the team, Which client should the charge nurse assign to the RN?a) An adolescent with multiple contusions due to a fall that occurred 2 days ago b) A 75-year-old with renal calculi who requires urine straining c) A 64-year- old client who had a total hip replacement the previous day d) A 30-year -old depressed client who admits to suicide ideations 7. A client who had a small bowel resection acquired methicillin resistant staphylococcus aureus (MRSA) while hospitalized. He treated and released but is readmitted today because of diarrhea and dehydration. It is most important for the nurse to implement which intervention. a) Maintain contact transmission precautions b) Instruct visitors to gown and wash hands. c) Collect serial stool specimens for culture d) Review white blood cell (WBC) count daily. 8. A preeclamptic client who delivered 24h ago remains in the labor and delivery recovery room. She continues to receive magnesium sulfate at 2 grams per hour. Her total input is limited to 125 ml per hour, and her urinary output for the last hour was 850 ml. What intervention should the nurse implement? a) discontinue the magnesium sulfate immediately b) Decrease the client's iv rate to 50 ml per hour c) Continue with the plan of care for this client d) Change the client's to NPO status 9. While a child is hospitalized with acute glomerulonephritis, the parents ask why blood pressure readings are taken so often. Which response by the nurse is most accurate? a) Blood pressure fluctuations means that the condition has become chronic b) Elevated blood pressure must be anticipated and identified quickly c) Hypotension leading to sudden shock can develop at any time d) Sodium intake with meals and snacks affects the blood pressure 10. A male client is admitted with a severe asthma attack. For the last 3 hours he has experienced increasing shortness of breath His arterial blood gas results are pH 7. 22, PaCO, 55 mmHg, HCO, 25 mEq/l or mmol/L. Which intervention should the nurse implement? a) Space care to provide periods of rest b) Instruct client to purse lip breathe c) Administer PRN dose of albuterol d) Position client for maximum comfort 11. An older adult client is admitted to the stroke unit after recovery from the acute phrase of an ischemic cerebral vascular accident (CAV). Which interventions should the nurse include in the plan of care during convalescence and rehabilitation? (Select all that apply.) a) Measure neurological vital signs every 4 hours b) Place a bed side commode next to bed c) Suction oral cavity every 4 hours. d) Encourage family participate in the client's care e) Play classical music in room while client is awake12. A female client with a history of heart failure (HF) arrives at the clinic after what she describes as a very long trip. Following the initial physical assessment and chart review, which prior action should the nurse implement? (click on each chart for additional information. Please be sure to scroll to the bottom right corner of each tab to view all information contained in the client's medical record) a) Reteach medication regimen b) Administer the prescribed diuretic c) Auscultate lung and heart sounds d) Give a potassium supplement.13. Which information is most important to include in the teaching plan for a client who is discharged after a thyroidectomy for Grave‟s disease? a) The need to use more blankets at night or extra clothing should be reported b) A high fiber diet with adequate fluid intake should be followed c) Meticulous eye care with artificial tears should be used daily d) Daily hormone replacement will be needed for the rest of the client’s life 14. The nurse is caring for four clients: Client A, who has emphysema and whose oxygen saturation is 94%; Client B, with a postoperative hemoglobin of 8.2 mg/dl; Client C, newly admitted with a potassium level of 3.8 mEq/L; and Client D scheduled for an appendectomy who has nurse implement? a white blood cell (WBC) count of 14,000 mm. What intervention should the nurse implement? a) Move client D into an isolation room 24 hour before surgery b) Verify that client B has two units of packed cells available c) Increase client A's oxygen to 4 liters a minute per cannula d) Ask the dietician to add a banana to Client C's breakfast tray 15. When administer ceftriaxone sodium intravenously to a client before surgery, which assessment finding requires the most immediate intervention by the nurse? a) Headache b) Pruritis c) Nausea d) Stridor 16. An older male client, who is a retired chef, is hospitalized with a diabetic ulcer on his foot His daughter tells the nurse that her father has become increasingly obsessed with the way his food is prepared in the hospital the nurse's response should be based on what information? a) The client probably has an organic brain disease and will likely have Alzheimer‟s disease within a few years. b) The family needs a social worker to talk to them about how to handle their father when he becomes annoying c) The daughter is under stress and should be encouraged to think about happier times d) The client was compulsive about food when he was younger, the aging process can magnify this 17. Following a motor vehicle collision (MVC), an unrestrained female client is admitted to the intensive care unit with altered mental status. She has multiple rib fractures and bruising across her lower abdomen. Which assessment finding warrants immediate intervention by the nurse? (Please scroll and view each tab's information in the client's medical record before selecting the answer) a) A large amount of gross hematuria b) Several apnea episodes lasting ten seconds c) Delayed peripheral capillary refill. d) Numbness of the left lower extremity 18. An older client comes to the clinic with a family member. When the nurse attempts to take the clients health history, the client does not respond to questions in a clear manner. What action should the nurse implement first? a) Assess the surroundings for noise and distractions b) Provide a printed health care assessment form c) Defer the health history until the client is less anxious d) Ask the family member to answer the question 19. The nurse enters a male client's room to administer oral medications and finds an unlicensed assistive personnel (UAP) providing personal care to the client. The client is lying in a supine position and his condition has obviously deteriorated. He is weak, pale, and diaphoretic. What is the priority nursing action? a) Determine why the UAP did not notify the nurse of the change in the client's condition b) Explain to the UAP that changes in a client's condition should be reported immediately c) Ask the UAP to position the client so the oral medications can be administered d) Advise the UAP to stop providing care so the nurse can assess the client's condition 20. A client with syndrome of inappropriate antidiuretic hormone (SIADH) is admitted with a weight gain of 4.4 lbs (2 kg) in 24 hours and an elevated pulse rate. Which intervention should the nurse implement first? a) Review arterial blood gas resultsb) Monitor daily sodium intake c) Record usual eating patterns d) Ensure client takes a diuretic every morning 21. The nurse implements a tertiary prevention program for type 2 diabetes in a rural health clinic. Which outcome indicates that the program was effective? a) Clients who developed disease complications promptly received rehabilitation b) More than 50% of at-risk clients were diagnosed early their disease process c) Only 30%of clients did not attend self-management education sessions d) Average client scores improved on specific risk factor knowledge test 22. A male client with cancer is admitted to the oncology unit and tells the nurse that he is in the hospital for palliative care measures. The nurse notes that the client‟s admission prescription includes radiation therapy. What action should the nurse implement? a) Ask the client about his expected goals for the hospitalization b) Explain the palliative care measures can be provided at home c) Notify do radiation department to withhold the treatment for now d) Determine if the client wishes to cancel further radiation treatment 23. The mother of a 7-month-old brings the infant to the clinic because the skin in the diaper area is excoriated and red, but there are no blisters or bleeding. The mother reports no evidence of watery stools. Which nursing intervention should the nurse implement? a) Instruct the mother to change the child’s diaper more often. b) Encourage the mother to apply lotion with each diaper charge c) Tell the mother to cleanse with soap and water at each diaper change d) Ask the mother to decrease the infant‟s intake of fruits for 24 hours. 24. A client with a serum sodium level of 125 meq/mL should benefit most from the administration of which intravenous solution? a) 0.9% sodium chloride solution (normal saline) b) 0.45% sodium chloride solution (half normal saline) c) 10% Dextrose in 0.45% sodium chloride d) 5% dextrose in 0.2% sodium chloride 25. What is the priority nursing problem for a client with hypoparathyroidism? a) Risk for injury. (because low calcium causes seizures) b) Deficient knowledge c) Anxiety d) imbalanced nutrition 26. A male client with cirrhosis and severe ascites, who is scheduled for a paracentesis tells the nurse that he is in pain and feels short of breath, so he wants to reschedule the procedure. How should the nurse respond? a) Encourage the client to verbalize his fears about the outcome of the procedure b) Offer to notify the healthcare provider of his desire to reschedule the procedure c) Advise the client that the procedure will help diagnose the cause of his symptoms d) Explain the client that the paracentesis will provide relief from his discomfort. 27. After having a pulmonary angiogram, a client is diagnosed with a pulmonary embolism (PE). Which intervention is most important for the nurse to include in the client's plan of care? a) Monitor for confusion and restlessness. (watching for brain embolism <stroke>) b) Observe for signs of increased bleeding. c) Administer IV opioids as needed for pain. d) Teach how to use incentive spirometry. 28. A client with chronic renal insufficiency is preparing for discharge from the hospital. Which information is most important for the nurse to include in this client's discharge teaching? a) Use of topical application to manage pruritus b) Need for maintaining good oral hygiene. c) Strategies to promote independent self-care. d) Instructions regarding a restricted protein diet (kidney can’t filter proteins) 29. The client provides three positive responses to items on the CAGE (cut down, Annoyed, Guilty, Eye-opener) Questionnaire. What interpretation should the nurse provide the client? a) The CAGE questionnaire is a tool used to identify general substance abuse b) At least two positive responses are strongly suggestive of alcohol dependence c) One positive response indicates the client should seek help with alcohol addiction d) All responses to the CAGE questionnaire must be positive to suggest alcohol dependence 30. A client is receiving ophthalmic drops preoperatively for a cataract extraction and asks the nurse why the healthcare provider has prescribed all these medications. Which information should the nurse included when responding to this client? (Select all that apply.) a) One of the medications is used to anesthetize the corneal surface. b) Pupillary dilation is necessary to access the eye chamber for lens removal. c) The iris must be paralyzed during surgery to prevent it from reacting to light. d) A medication is used to induce sleep during the procedure. e) These medications assist in obstructing client´s vision during the surgery. 31. The nurse observes a practical nurse (PN) pouring warm water over the perineal area of a female client who has frequent urinary incontinence while the client is positioned on a bedpan. What action should the nurse take? a) Recommend a complete bath to cleanse the perineal area more fully b) Suggest contacting the healthcare provider for a prescription for catheter insertion c) Evaluate the effectiveness of this measure to client d) Instruct the PN that this technique promotes infection in elderly females 32. The healthcare provider prescribes amoxicillin 1.5 grams PO daily, in equaly divided doses to be administered every 8 hours. The medication is available in a bottle labeled “Amoxicillin suspension 200mg/5ml”. How many ml should the nurse administer every 8 hours? (Enter numeric value only. If rounding is required, round to the nearest tenth). 12.533. The nurse should expect a client diagnosed with regional enteritis (Crohn's disease) to exhibit what initial symptoms? a) Diarrhea, abdominal pain, and weight loss b) Change in bowel habits, blood in stool, and unexplained anemia c) Dull, left lower cramping pain and low grade fever. d) Rigid board-like abdomen and elevated white blood cell count. 34. The nurse is interacting with a female client who is diagnostic with postpartum depression. Which findings should the nurse document as an objective signs of depression? (Select all that apply) a) Expresses suicidal thoughts b) Avoid eyes contact c) Reports feeling sad d) Has a disheveled appearance e) Interacts with felt effect 35. When conducting diet teaching for a client who is on a postoperative full liquid diet, which foods should the nurse encourage the client eat? (Select all that apply.) a) Cheese b) Lentils c) Tea d) Whole grain breads e) Potato soup 36. A client who is hospitalized and recently diagnosed with Addison‟s disease is now confused and lethargic. Which actions should the nurse implement? (Select all that apply) a) Measure capillary glucose level. b) Monitor cardiac telemetry pattern. c) Reduce rate of intravenous fluid infusion. d) Withhold next dose of corticosteroid. e) Initiate fall risk precautions. 37. After removing a client‟s dressing that is saturated with sanguineous drainage, where should the nurse place the dressing? a. Red-bag container b. Sharp container c. Non-permeable pad d. Sink 38. Immediately after extubating, a client who has been mechanically ventilated is placed on a 50% non-rebreather. The client is hoarse and complaining of a sore throat. Which assessment finding should the nurse report to the healthcare provider immediately? a) Blood tinged sputum b) Expiratory wheezing c) Upper airway stridor (laryngomalasia) d) Oxygen saturations 90%39. The nurse manager is involved in agency restructuring. During this re-engineering process, it is most important for the nurse to address which employee concern? a) Employees' job security b) New management's expectations c) Potential changes in employee benefits d) Changes in job descriptions. 40. An adult client presents to the clinic with a large draining ulcers on both lower legs that are characteristic of Kaposi´s Sarcoma lesions. The client is accompanied by two family members. What action should the nurse take? a) Obtain a blood sample to determine if the client is HIV positive b) Send family to the waiting area while the clients history is taken c) Complete a head to toe assessment to identify other signs of HIV d) Ask the family members to wear gloves when touching the client 41. Prior to teaching a client how to walk with crutches, which exercises are most important for the nurse to encourage the immobilized client to practice? a) Hip abduction and adduction b) Shoulder shrugs and rotation c) Body lifts using the arms d) Head and chin rolls 42. A female client comes to the emergency center to report that two days ago she was the victim of date rape. Which action is most important for the nurse to implement? a) Assess for the presence of serum flunitrazepam b) Inquire if recent consensual sexual encounters have occurred c) Sign across the sealed openings of evidence envelopes d) Place collected specimens in separate envelopes 43. An older client is referred to a rehabilitation facility following a cerebrovascular accident is with left-side paresis and is having difficulty swallowing. Which intervention is most important for the nurse to include in the client's plan of care? a) Initiate passive range of motion exercises. b) Arrange for daily home care assistance c) Facilitate a consultation for speech therapy d) Use pictures and gestures to communicate. 44. The nurse is teaching a male adolescent recently diagnosed with type 1 diabetes mellitus (DM) about self-injecting insulin. Which approach is best for the nurse to use to evaluate the effectiveness of the teaching? a) Observe him as he demonstrates the self-injection technique to another diabetic adolescent b) Ask the adolescent to describe his level of comfort with injecting himself with insulin c) Have the adolescent list the procedural steps for safe insulin administration d) Review his glycosylated hemoglobin level 3 months after the teaching session45. Which client is best to assign to the LPN who is assisting the RN with the care of a group of clients? a) An older adult who is scheduled for foot amputation due to diabetes complications. b) An adult with alcoholism, cirrhosis, and hepatic encephalopathy c) An older client who is one day postoperative with a colostomy for colon cancer d) An adult who is one day postoperative for a laparoscopic cholecystectomy 46. A confused, older client with Alzheimer‟s disease becomes incontinent of urine when attempting to find the bathroom. Which action should the nurse implement? a) Instruct the client to use the call button when a bedpan is needed b) Apply adult diapers after each attempt to void c) Check residual urine volume using an indwelling urinary catheter d) Assist the client’s to a bedside commode every two hours 47. A client with C-6 spinal cord injury rehabilitation. In the middle of the night the client reports a severe, pounding headache, and has observable piloerection or “goosebumps”. The nurse should asses for which trigger? a) Loud hallway noise. b) Fever c) Full bladder d) Frequent cough. 48. After inflating a blood pressure cuff and releasing the valve, the nurse hears silence followed by a Korotkoff sound. What action should the nurse take next? a) Reinflate the cuff to a higher number b) Reposition the stethoscope over the brachial artery c) Continue with the blood pressure assessment d) Note the presence of a auscultatory gap. 49. The nurse reviews the laboratory results for a client who is admitted with cholelithiasis and identifies the presence of urobilinogen in the urine analysis and elevated serum values for indirect bilirubin, direct bilirubin, and total bilirubin. Based on these findings, which nursing intervention would be considered the priority to include in this client's plan care? a) Implement bleeding precautions. b) Give analgesics as prescribed for biliary colic c) Document changes in stool color d) Provide skin care for pruritus 50. A client is to receive mannitol (osmitrol) intravenously for increased intracranial pressure. Which outcome may result from the use of? a) Hyponatremia b) Hypervolemia c) Oliguria d) Hyperglycemia51. A young adult client is admitted to the emergency room following a motor vehicle collision. The client‟s head hit the dashboard. Admission assessment include: Blood pressure 85/45 mm Hg, temperature 98.6 F, pulse 124 beat/minute and respirations 22 breath/minute. Based on these data, the nurse formulates the first portion of nursing diagnosis as “Risk of injury”. What term best expresses the “related to” portion of nursing diagnosis? a) Infection b) Increase intracranial pressure c) Shock d) Head Injury 52. When caring for a client who has acute respiratory distress syndrome (ARDS), the nurse elevates the head of the bed 30 degrees. What is the reason for this intervention? a) To reduce abdominal pressure on the diaphragm b) to promote retraction of the intercostal accessory muscle of respiration c) to promote bronchodilation and effective airway clearance d) to decrease pressure on the medullary center which stimulates breathing 53. A nurse who usually works on step-down unit is moved to work a 12-hour shift in the critical care unit. Which client is best for the charge nurse to assign to this care? a) A ventilated client admitted today with respiratory failure acidosis. b) A ventilator dependent client with chronic obstructive pulmonary disease (COPD). c) A client admitted for a narcotic overdose who is ventilated and has respiratory alkalosis. d) A client who has a new onset diabetic ketoacidosis (DKA) and is on an insulin drip. 54. The nurse enters a male client‟s room to administer oral medications and find an unlicensed assistive personnel (UAP) providing personal care to the client. The client is lying is a supine position and his condition has obviously deteriorated. He is weak, pale, and diaphoretic. What is the priority nurse action? a) Determine why the UAP did not notify the nurse of the change in the client‟s condition. b) Ask the UAP to position the client so the oral medications can be administered. c) Advise the UAP to stop providing care so the nurse can assess the client’s condition. d) Explain to the UAP that changes in a client‟s condition should be reported immediately. 55. A preeclamptic client who delivered 24 hours ago remains in the labor and delivery recovery room. She continues to receive magnesium sulfate at 2 grams per hours. Her total input is limited to 125 ml per hours, and her urinary output for the last hour was 850 ml. What intervention should the nurse implement? a) Decrease the client‟s IV rate to 50 ml per hour. b) Change the client‟s diet to NPO status. c) Continue with the plan of care for this client. d) Discontinue the magnesium sulfate immediately. 56. The nurse is caring for four clients: Client A, who has emphysema and whose oxygen saturation is 94%; Client B with a postoperative hemoglobin of 8.7 mg/ di ;Client C, newly admitted with a potassium level of 3.8 mEq/ L; and Client D, scheduled for an appendectomy who has a white blood cell count of 15.000 mm3. What intervention should the nurse implement? a) Move client D into isolation room 24 hours before surgery b) Increase Client A‟s oxygen to 4 liters a minute per cannula c) Verify that Client B has two units of packed cells available d) Ask the dietician to add banana to Client C‟s breakfast tray 57. .A client with a new diagnosis of Raynaud´s disease lives alone. Which instruction should the nurse include in this client´s discharge teaching plan? a) Wear TED stockings at night. b) Hire a care-giver for 8 hours daily. c) Keep room temperature 80 F. d) Develop a walking exercise routine 58. An adult client with a broken femur is transferred to the medical surgical unit to await surgical internal fixation after the application of an external traction device to stabilize the leg. An hour after an opioid analgesic was administered, the client report muscle spasms and pain at the fracture site. While waiting for the client to be transported to surgery, which action should the nurse implement!? a) Observe for signs of deep vein thrombosis. b) Administer PRN dose of a muscle relaxant. c) Check client´s most recent electrolyte values. d) Reduce the weight on the traction device. 59. A client diagnosed with calcium kidney stones has a history of gout. A new prescription for aluminum hydroxide (Amphogel) is scheduled to begin at 0730. Which client medication should the nurse bring to the healthcare provider‟s attention? a) Allopurinol (Zyloprim) b) Aspirin, low dose c) Furosemide (lasix) d) Enalapril (vasote) 60. 61-An older client is admitted for repair of a broken hip. To reduce the risk for infection in the postoperative period, which nursing care interventions should the nurse include in the client‟s plan of care? (Select all that apply) a) Teach client to use incentive spirometer q2 hours while awake. b) Remove urinary catheter as soon as possible and encourage voiding. c) Maintain sequential compression devices while in bed. d) Administer low molecular weight heparin as prescribed e) Assess pain level and medicate PRN as prescribed. 61. The nurse notes the client receiving heparin infusion labeled, Heparin Na 25,000 Units in 5% Dextrose injection 500 ml at 50ml/hr. What dose of Heparin is the client receiving per hour? 2500 U62. An unlicensed assistive personnel UAP leaves the unit without notifying the staff. In what order should the unit manager implement this intervention to address the UAPs behavior? (Place the action in order from first on top to last on bottom.) a) Note date and time of the behavior. b) Discuss the issue privately with the UAP. c) Plan for scheduled break times. d) Evaluate the UAP for signs of improvement. 63. 54-A male client is admitted with a severe asthma attack. For the last 3 hours he has experienced increased shortness of breath. His arterial blood gas results are: pH 7.22 PaCO2 55 mmHg; HCO3 25 mEq/L or mmol/L (SI). Which intervention should the nurse implement? a) Space care to provide periods of rest b) Instruct client to purse lip breathe c) Administer PRN dose of albuterol d) Position client for maximum comfort 64. A client hospitalized with a fractured mandible is to discharge. Which equipment should the client be instructed to have available at all times? a) Oral airway b) Screw driver c) Tracheostomy set. d) Wire cutters. 65. During a Women‟s Health Fair, which assignment is best for the practical nurse (PN) who is working with a registered nurse (RN)? a) Encourage a woman at risk for cancer to obtain a colonoscopy. b) Prepare a woman for a bone density screening. c) Present a class on breast self-examination. d) Explain the follow-up needed for a client with prehypertension. 66. The nurse notes that an older adult client has a moist cough that increase in severity during and after meals. Based on this finding, what action should the nurse take. a) Encourage client to deep breathing exercises daily. b) Offer the client additional clear frequently. c) Collect a sputum specimen immediately. d) Request a consultation to confirm dysphagia 67. The nurse mixes 250 mg of debutamine in 250 ml of D5W and plans to administer the solution at rate 15mcg/kg/minute a client weighing 110 pounds. The nurse should set the infusion pump to administer how many ml per hour only. If rounding is required, round the nearest whole number.) 45 68. A 16-year-old adolescent with meningococcal meningitis is receiving a continuous IV infusion of penicillin G, which is prescribed as 20 million units in a total volume of 2 liters of normal saline every 24 hr. The pharmacy delivers 10 million units/ liters of normal saline. How many ml/hr should the nurse program the infusion pump? (Enter numeric value only. If rounding is required, round to the nearest whole number.) Answer 8369. A client in the third trimester of pregnancy reports that she feels some “lumpy places “ in her breast and that her nipples sometimes leak yellowish fluid. She has an appointment with her healthcare provider I two weeks. What action should the nurse take? a) Reschedule the client‟s prenatal appointment for the following da b) Recommend that the client start wearing a supportive brassiere c) Tell the client to begin nipple stimulation to prepare for breast feeding d) Explain that this normal secretion can be assessed at the nest visit 70. An older adult client admitted to the stroke unit after recovery from the acute phrase of an ischemic cerebral vascular accident (CVA). Which intervention should the nurse include in the plan of care during convalescence and rehabilitation? (Select all that apply.) a) Measure neurological vital signs every 4 hours b) Place a bedside commode next to the bed c) Suction oral cavity every 4 hours d) Encourage family to participate in the client’s care e) Play classical music in room while client is awake 71. An older male client, who is a retired chef, is hospitalized with a diabetic ulcer on his foot. His daughter tells the nurse that her father has become increasingly obsessed with the way food is prepared in the hospital the nurse‟s response should be based on what information? a) The client probably has an organic brain disease and will likely have Alzheimer‟s disease within a few years. b) The family needs a social worker to talk to them about how to handle their father when he becomes annoying. c) The daughter is under stress and should be encourage too think about happier times. d) If the client was compulsive about food when he was younger, the aging process can magnify this. 72. The nurse is preparing a client with as acoustic neuroma for a magnetic resonance image (MRI). Which client complaint is life threatening and should be reported to the healthcare provider immediately? a) Intensifying headache b) Facial numbness c) Difficulty with balance d) Right ear hearing loss 73. 66-A client who is at 10-weeks‟ gestation calls the clinic because she has been vomiting for the past 24 hours. The nurse determined that the client has no fever. Which instruction should the nurse give this client? a) Make an appointment at he clinic if a fever occurs b) Remain on clear liquids until the vomiting subsides c) Take nothing by mouth until there is no more nausea d) Come to the clinic to be seen by the healthcare provider 74. The nurse should expect a client diagnosis with regional enteritis (Crohn‟s disease) to exhibit what initial symptoms?a) Diarrhea, abdominal pain, and weight loss b) Change in bowels habits, blood in stool, and unexpected anemia c) Dull, left lower cramping pain and low grade fever d) Rigid board-like abdomen and elevated white blood cell count 75. An unresponsive male victim of a diving accident is brought to the emergency department where it is determining that immediate surgery is required to save his life. The client is accompanied by a close friend, but no family members are available. What action should the nurse take first. a) Ask he man‟s friend to sign the informed consent since the client is unresponsive b) Notify the unit manager that an emergency court order is needed to allow surgery. c) Continue to provide the support until a thorough search for a guardian is completed. d) Carry on with surgical preparation of the client without a signed informed consent. 76. A 300 ml unit of packed red blood cells is prescribed for a client with heart failure (HF) who has 3+ pitting edema, shortness of breath with any activity, and crackles in both lung bases. What rate should the nurse administer the blood? a) 150 ml/hour b) 50 ml/hr c) 300 ml/hr d) 75 ml/hr 77. An older female client with osteoarthritis reports increasing pain and stiffness in her right knee and asks how to reduce these symptoms. In responding to the client, the nurse recognizes what pathology as the cause of her symptoms? a) Systematic inflammatory response b) Loss of bone mineral density c) Infectious process in the synovial fluid d) Destruction of joint cartilage 78. The nurse is caring for a newborn who arrives in the nursery following a precipitous birth on the way to the hospital. A drug screen of the mother reveals the presence of cocaine metabolites. The infant has a heart rate of 175 beats/ minute, cries continuously, is irritable, and is hyperreactive to stimuli. Which intervention is most important for the nurse to include in this infant‟s plan of care? a) Initiate infant sepsis protocol b) Implements seizure precautions c) Refer to protective child services d) Formula feed every 3 hours 79. Before leaving the room of a confused client, the nurse notes that a half bow knot was used to attach the client's wrist restraints to the movable portion of the client's bed frame. What action should the nurse take before leaving the room? a) Ensure that the knot can be quickly released. b) Tie the knot with a double turn or square knot. c) Move the ties so the restraints are secured to the side rails.d) Ensure that the restraints are snug against the client's wrist. 80. The nurse notes that a depressed female client has been more withdrawn and noncommunicative during the past two weeks. Which intervention is most important to include in the updated plan of care for this client? a) Encourage the client‟s family to visit more often b) Schedule a daily conference with the social worker c) Encourage the client to participate in group activities d) Engage the client in a non-threatening conversation. 81. An adolescent girl with anorexia nervosa is within 10 pounds of her normal weight and is being discharge from the mental health unit with a prescription for fluoxetine (prozac). Which instruction is most important for the nurse to provide the parent listen for the daughter expressions of wanting to harm herself a) Listen for the adolescent’s expressions of wanting to harm themselves b) Check the adolescent‟s mouth to ensure that they had swallowed fluoxetine c) Observe their adolescent weight themselves at the same time every day d) Encourage activities that allow their adolescent to exhibit control 82. A client was admitted to the cardiac observation unit 2 hours ago complaining of chest pain. On admission, the client‟s EKG showed bradycardia, ST depression, but no ventricular ectopy. The client suddenly reports a sharp increase in pain, telling the nurse, “I feel like an elephant just stepped on my chest” The EKG now shows Q waves and ST segment elevations in the anterior leads. What intervention should the nurse perform? a-Increase the peripheral IV flow rate to 175 ml/hr to prevent hypotension and shock b-Administer prescribed morphine sulfate IV and provide oxygen at 2 L/min per nasal cannula. c- Obtain a stat 12 lead EKG and perform a venipuncture to check cardiac enzymes levels. d-Notify the healthcare provider of the client‟s increase chest pain a call for the defibrillator crash cart. 83. To reduce the risk of symptoms exacerbation for a client with multiple sclerosis (MS), which instructions should the nurse include in the client‟s discharge plan? (Select all that apply). a) Practice relaxation exercises b) Limit fluids to avoid bladder distention c) Space activities to allow for rest periods d) Avoid persons with infections e) Take warm baths before starting exercise 84. Three hours following a right carotid endarterectomy, the nurse notes a moderate amount of bloody drainage on the client‟s dressing. Which additional assessment finding warrants immediate intervention by the nurse. a) Sore throat when swallowing b) Tongue deviation to the left c) Palpable temporal pulses d) Temperature of 99.2F (37.3 C)85. A client who is admitted to the intensive care unit with syndrome of inappropriate antidiuretic hormone (SIADH) has developed osmotic demyelination. Which intervention should the nurse implement first? a) Patch one eye. b) Reorient often. c) Range of motion. d) Evaluate swallow 86. The nurse is planning to assess a client's oxygen saturation to determine if additional oxygen is needed via nasal cannula. The client has a bilateral below-the-knee amputation and pedal pulses that are weak and threaty. What action should the nurse take? a) Document that an accurate oxygen saturation reading cannot be obtained b) Elevate to client's hands for five minutes prior to obtaining a reading from the finger c) Increase the oxygen based on the clients breathing patterns and lung sounds d) Place the oximeter clip on the ear lobe to obtain the oxygen saturation reading 87. The nurse is completing a neurological assessment on a client with a closed head injury. The Glasgow Coma Scale (GCS) score was 13 on admission. It is now assessed at 6. What is the priority nursing intervention based on the client´s current GCS? a) Notify the healthcare provider of the GCS score b) Prepare the family for the client‟s imminent death c) Monitor the client q1 hour for changes in the GCS score d) Begin cardiopulmonary resuscitation (CPR) 88. A client with a history of dementia has become increasingly confused at night and is picking at an abdominal surgical dressing and the tape securing the intravenous (IV) line. The abdominal dressing is no longer occlusive, and the IV insertion site is pink. What intervention should the nurse implement? a) Replace the IV site with a smaller gauge. b) Redress the abdominal incision c) Leave the lights on in the room at night. d) Apply soft bilateral wrist restraints. 89. A 3-year-old boy was successfully toilet trained prior to his admission to the hospital for injuries sustained from a fall. His parents are very concerned that the child has regressed in his toileting behaviors. Which information should the nurse provide to the parents? a) Children usually resume their toileting behaviors when they leave the hospital b) A retraining program will need to be initiated when the child returns home c) Diapering will be provided since hospitalization is stressful to preschoolers d) A potty chair should be brought from home so he can maintain his toileting skills 90. A client is receiving lactulose (Portalac) for signs of hepatic encephalopathy. To evaluate the client's therapeutic response to this medication, which assessment should the nurse obtain? a) Level of consciousness. b) Percussion of abdomenc) Serum electrolytes d) Blood glucose. 91. A 13 years-old girl, diagnosed with DM Type 1 at the age of 9, is admitted to the hospital in diabetic ketoacidosis. Which occurrence is the most likely cause of the ketoacidosis? a) Ate an extra peanut butter sandwiches before gym class b) Incorrectly drew up and administered too much insulin c) Was not hungry, so she skipped eating lunch d) Has had a cold and ear infection for the past two days 92. What instruction should the nurse provide to a client who is preparing to have a cystoscopy? a) Avoid strenuous activity and sports for a least 2 weeks b) Report any allergies to shellfish or iodine c) Lay prone for 24 hours after the procedure d) Report any painful urination, blood urine, or fever 93. On admission to the Emergency Department, a female client who was diagnosed with bipolar disorder 3 years ago reports that this morning she took a handful of medications and left a suicide note for her family. Which information is most important for the nurse to obtain? a) Which family member has the clients suicide note b) When the client las took drugs for bipolar disorder c) What drugs the client used for the suicide attempt d) Whether the client ever attempted suicide in the past 94. A nurse that she is concerned about her 81 – year old neighbor, a widow whose son recently assumed her financial affairs. Lately, her neighbor has become reclusive, but is occasionally seen walking outside wearing only robe and slippers. What response should the nurse offer? a) Explain that it is not unusual for older adults to suffer from dementia which often causes such behaviors b) Tell the client to talk to a healthcare provider before reporting suspicion of neglect to the authorities c) Provide the number for Adult Protective Services so the client can report any suspicion of elder abuse d) Encourage the client to avoid becoming involved the neighbor‟s problems, for one‟s own protection 95. While changing the dressing of a client who is immobile, the nurse notices the boundary of the wound has increased. Before reporting this finding to the healthcare provider, the nurse should evaluate which of the client‟s laboratory values? a) Serum potassium and sodium levels. b) C-reactive protein level. c) Platelet count. d) Neutrophil count.96. While changing a client‟s chest tube dressing, the nurse notes a crackling sensation when gentle pressure is applied to the skin at the insertion site. What is the best action for the nurse to take? a-Apply a pressure dressing around the chest tube insertion site. b-Assess the client for allergies to topical cleaning agents. c-Measure the area of swelling and crackling. d-Administer an oral antihistamine per PRN protocol. 97. Which assessment finding is most important when planning to provide a complete bed bath to a bedfast client? a) 2+ pitting edema of the feet b) Right-side paralysis c) Orthopnea d) Pallor 98. A small, round raised are appears under the client‟s skin as the nurse administers an intradermal medication. What actions should the nurse take? a) Apply a col pack to the area for twenty minutes b) Elevate the area and apply light pressure over the site c) Document the site where the medication was given d) Notify the healthcare provider of the allergic response 99. The nurse instructs UAP to turn an immobilized elderly client with an indwelling urinary catheter every two hour. What additional action should the nurse instruct the UAP to take each time the client is turned? a) Empty the urinary drainage bag b) Feed the client a snack c) Offer the client oral fluids d) Assess the breath sounds 100. A heparin infusion is prescribed for a client who weighs 220 pounds after administering a bolus dose of 80 units/kg. The nurse calculates the infusion rate for the heparin sodium at 18 units/kg/hour. The available solution is Heparin Sodium 25,000 units in 5% Dextrose Injection 250 ml. The nurse should program the infusion pump to deliver how many ml/hour. 18 101. At 1615, prior to ambulating a postoperative client for the first time, the nurse reviews the client‟s medical record. Based on date contained in the record, what action should the nurse take before assisting the client with ambulation:a) Remove sequential compression devices. b) Apply PRN oxygen per nasal cannula. c) Administer a PRN dose of an antipyretic. d) Reinforce the surgical wound dressing. 102. A female client presents in the Emergency Department and tells the nurse that she was raped last night. Which question is most important for the nurse to ask? a-Does she knows the person who raped her? b-Has she taken a bath since the raped occurred? c-Is the place where she lived a safe place? d-Did she report the rape to the police Department? 103. The school nurse is screening students for spinal abnormalities and instructs each student to stand up and then touch the toes. Which finding indicates a student should be referred for scoliosis evaluation? a) Inability to touch toes b) Asymmetry of the shoulders when standing upright c) Audible crepitus when bending d) An exaggerated upper thoracic convex curvature 104. Which client should the nurse assess frequently for risk of overflow incontinence? a) Client who is bedfast, with increased serum BUN and creatinine levels b) Client with hematuria and decreasing hemoglobin and hematocrit levels c) Client who has a history of frequent urinary tract infections d) Client who is confused and frequently forgets to go to the bathroom 105. A male client with hypertension, who received new antihypertensive prescriptions at his last visit returns to the clinic two weeks later to evaluate his blood pressure (BP). His BP is 158/106 and he admits that he has not been taking the prescribed medication because the drugs make him “feel bad”. In explaining the need for hypertension control, the nurse should stress that an elevated BP places the client at risk for which pathophysiological condition? Stroke secondary to hemorrhage 106. The charge nurse of a critical care unit is informed at the beginning of the shift that less than the optimal number of registered nurses will be working that shift. In planning assignments, which client should receive the most care hours by a registered nurse (RN)? An 82-year-old client with Alzheimer’s disease newly-fractures femur who has a Foley catheter and soft wrist restrains applied 107. A client with multiple sclerosis (MS) has decreased motor function after taking a hot bath (Uhthoff‟s sign). Which pathophysiological mechanism supports this response? Temporary vasodilation 108. During the admission assessment, the nurse auscultates heart sounds for a client with no history of cardiovascular disease. Where should the nurse listen when assessing the client‟s point of maximal impulse (PMI) (Click the chosen location. To change, click on a new location) 4th left intercostal space 109. Which intervention should the nurse include in the plan of care for a child with tetanus? Minimize the amount of stimuli in the room 110. A client with leukemia undergoes a bone marrow biopsy. The client‟s laboratory values indicate the client has thrombocytopenia. Based on this data, which nursing assessment is most important following the procedure? a) Observe aspiration site. b) Assess body temperature c) Monitor skin elasticity d) Measure urinary output 111. During discharge teaching, the nurse discusses the parameters for weight monitoring with a client who was recently diagnosed with heart failure (HF). Which information is most important for the client to acknowledge? Report weight gain of 2 pounds (0.9kg) in 24 hours 112. The public nurse health received funding to initiate primary prevention program in the community. Which program the best fits the nurse‟s proposal? a) Case management and screening for clients with HIV. b) Regional relocation center for earthquake victims c) Vitamin supplements for high-risk pregnant women. d) Lead screening for children in low-income housing. 113. An elderly male client is admitted to the mental health unit with a sudden onset of global disorientation and is continuously conversing with his mother, who died 50 years ago. The nurse reviews the multiple prescriptions he is currently taking and assesses his urine specimen, which is cloudy, dark yellow, and has foul odor. These findings suggest that his client is experiencing which condition? a) Delirium b) Depression c) Dementia d) Psychotic episode 114. Which intervention should the nurse implement during the administration of vesicant chemotherapeutic agent via an IV site in the client‟s arm? Assess IV site frequently for signs of extravasation 115. When development a teaching plan for a client newly diagnosed type 1 diabetes, the nurse should explain that an increase thirst is an early sing of diabetes ketoacidosis (DKA), which action should the nurse instruct the client to implement if this sign of DKA occur? a) Resume normal physical activity b) Drink electrolyte fluid replacement c) Give a dose of regular insulin per sliding scale d) Measure urinary output over 24 hours. Rationale: As hyperglycemia persist, ketone body become a fuel source, and the client manifest early signs of DKA that include excessive thirst, frequent urination, headache, nausea and vomiting. Which result in dehydration and loss of electrolyte. The client should determine fingersticks glucose level and self-administer a dose of regular insulin per sliding scale. 116. An unlicensed assistive personnel (UAP) reports that a client‟s right hand and fingers spasms when taking the blood pressure using the same arm. After confirming the presence of spams what action should the nurse take? a) Ask the UAP to take the blood pressure in the other arm b) Tell the UAP to use a different sphygmomanometer. c) Review the client’s serum calcium level d) Administer PRN antianxiety medication. Rationale: Trousseau’s sign is indicated by spasms in the distal portion of an extremity that is being used to measure blood pressure and is caused by hypocalcemia (normal level 9.0-10.5 mg/dl, so C should be implemented 117. A client is being discharged home after being treated for heart failure (HF). What instruction should the nurse include in this client‟s discharge teaching plan? a) Weigh every morning b) Eat a high protein diet c) Perform range of motion exercises d) Limit fluid intake to 1,500 ml daily 118. The nurse notes a depressed female client has been more withdrawn and noncommunicative during the past two weeks. Which intervention is most important to include in the updated plan of care for this client? a) Encourage the client‟s family to visit more often b) Schedule a daily conference with the social worker c) Encourage the client to participate in group activities d) Engage the client in a non-threatening conversation. Rationale: Consistent attempts to draw the client into conversations which focus on non-threatening subjects can be an effective means of eliciting a response, thereby decreasing isolation behaviors. There is not sufficient data to support the effectiveness of A as an intervention for this client. Although B may be indicated, nursing interventions can also be used to treat this client. C is too threatening to this client. 119. The nurse is assessing an older adult with type 2 diabetes mellitus. Which assessment finding indicates that the client understands long- term control of diabetes? a) The fating blood sugar was 120 mg/dl this morning. b) Urine ketones have been negative for the past 6 months c) The hemoglobin A1C was 6.5g/100 ml last week d) No diabetic ketoacidosis has occurred in 6 months. Rationale: A hemoglobin A1C level reflects he average blood sugar the client had over the previous 2 to 3 month, and level of 6.5 g/100 ml suggest that the client understand long-term diabetes control. Normal value in a diabetic patient is up to 6.5 g/100 ml. 120. An adult male client is admitted to the emergency room following an automobile collision in which he sustained a head injury. What assessment data would provide the earliest that the client is experiencing increased intracranial pressure (ICP)? a) Lethargy b) Decorticate posturing c) Fixed dilated pupil d) Clear drainage from the ear.Rationale: Lethargy is the earliest sign of ICP along with slowing of speech and response to verbal commands. The most important indicator of increase ICP is the client’s level or responsiveness or consciousness. B and C are very late signs of ICP. 121. An older adult male is admitted with complications related to chronic obstructive pulmonary disease (COPD). He reports progressive dyspnea that worsens on exertion and his weakness has increased over the past month. The nurse notes that he has dependent edema in both lower legs. Based on these assessment findings, which dietary instruction should the nurse provide? a) Limit the intake of high calorie foods. b) Eat meals at the same time daily. c) Maintain a low protein diet. d) Restrict daily fluid intake. Rationale: the client is exhibiting signs of cor pulmonale, a complication of COPD that causes the right side of the heart to fail. Restricting fluid intake to 1000 to 2000 ml/day, eating a high-calorie diet at small frequent meals with foods that are high in protein and low in sodium can help relive the edema and decrease workload on the right-side of the heart. 122. An older male client arrives at the clinic complaining that his bladder always feels full. He complains of weak urine flow, frequent dribbling after voiding, and increasing nocturia with difficulty initiating his urine stream. Which action should the nurse implement? a) Obtain a urine specimen for culture and sensitivity b) Palpate the client's suprapubic area for distention c) Advise the client to maintain a voiding diary for one week d) Instruct in effective technique to cleanse the glans penis Rationale: the client is exhibiting classic signs of an enlarge prostate gland, which restricts urine flow and cause bothersome lower urinary tract symptoms (LUTS) and urinary retention, which is characterized by the client's voiding patterns and perception of incomplete bladder emptying. 123. The nurse is preparing an older client for discharge following cataract extraction. Which instruction should be include in the discharge teaching? a) Do not read without direct lighting for 6 weeks. b) Avoid straining at stool, bending, or lifting heavy objects. c) Irrigate conjunctiva with ophthalmic saline prior to installing antibiotic ointment. d) Limit exposure to sunlight during the first 2 weeks when the cornea is healing. Rationale: after cataract surgery, the client should avoid activities which increase pressure and place strain on the suture line. 124. Assessment by the home health nurse of an older client who lives alone indicates that client has chronic constipations. Daily medications include furosemide for hypertension and heart failure and laxatives. To manage the client‟s constipation, which suggestions should the nurse provide? (Select all that apply) a) Decrease laxative use to every other day, and use oil retention enemas as needed. b) Include oatmeal with stewed pruned for breakfast as often as possible. c) Increase fluid intake by keeping water glass next to recliner. d) Recommend seeking help with regular shopping and meal preparation. e) Report constipation to healthcare provider related to cardiac medication side effects.Rational: older adult are at higher risk for chronic constipation due to decreased gastrointestinal muscle tone leading to reduce motility. Oatmeal with prunes increases dietary fiber and bowel stimulation, thereby decreasing need for laxatives. Increased fluid intake also decreases constipations. Assistance with food preparation might help the client eat more fresh fruits and vegetables and result on less reliance on microwaved and fast foods, which are usually high in sodium and fat with little fiber. Laxatives can be reduced gradually by improving the diet, without resorting to using enemas. 125. A client with osteoporosis related to long-term corticosteroid therapy receives a prescription for calcium carbonate. Which client‟s serum laboratory values requires intervention by the nurse? a) Total calcium 9 mg/dl (2.25 mmol/L SI) b) Creatinine 4 mg/dl (354 micromol/L SI) c) Phosphate 4 mg/dl (1.293 mmol/L SI) d) Fasting glucose 95 mg/dl (5.3 mmol/L SI) 126. A client is admitted with acute pancreatitis. The client admits to drinking a pint of bourbon daily. The nurse medicates the client for pain and monitors vital signs q2 hours. Which finding should the nurse report immediately to the healthcare provider? a) Confusion and tremors b) Yellowing and itching of skin. c) Abdominal pain and vomiting d) Anorexia and abdominal distention Rationale: daily alcohol is the likely etiology for the client’s pancreatitis. Abrupt cessation of alcohol can result in delirium tremens (DT) causing confusion and tremors, which can precipitate cardiovascular complications and should be reported immediately to avoid life-threatening complications. The other options are expected findings in those with liver dysfunction or pancreatitis, but do not require immediate action 127. After receiving report, the nurse can most safely plan to assess which client last? The client with… a) A rectal tube draining clear, pale red liquid drainage b) A distended abdomen and no drainage from the nasogastric tube c) No postoperative drainage in the Jackson-Pratt drain with the bulb compressed d) Dark red drainage on a postoperative dressing, but no drainage in the Hemovac®. Rationale: The most stable client is the one with a functioning drainage device and no drainage. This client can most safely be assesses last. Other clients are either actively bleeding, have an obstruction in the nasogastric tube which may result in vomiting, or may be bleeding and / or may have a malfunction in the Hemovac® drain. 128. A client is receiving lactulose (Portalac) for signs of hepatic encephalopathy. To evaluate the client‟s therapeutic response to this medication, which assessment should the nurse obtain? a) Level of consciousness b) Percussion of abdomen c) Serum electrolytes d) Blood glucose. Rationale: Colonic bacteria digest lactulose to create a drug-induces acidic and hyperosmotic environment that draws water and blood ammonia into the colon and coverts ammonia to ammonium, which is trapped in the intestines and cannot be reabsorbed into the systemic circulation. This therapeutic action of lactulose is to reduce serum ammonia levels, which improves the client’s level of consciousness and metal status. 129. A female nurse who took drugs from the unit for personal use was temporarily released from duty. After completion of mandatory counseling, the nurse has asked administration to allow her to return to work. When the nurse administrator approaches the charge nurse with the impaired nurse request, which action is best for the charge nurse to take? a) Since treatment is completed, assign the nurse to the route RN responsibilities b) Ask to meet with impaired nurse‟s therapist before allowing her back on the unit. c) Allow the impaired nurse to return to work and monitor medication administration d) Meet with staff to assess their feelings about the impaired nurse‟s return to the unit. Rationale: provides essential monitoring and helps ensure nurse compliance and promote client safety. 130. The nurse is teaching a male adolescent recently diagnosed with type 1diabetes mellitus (DM) about self-injecting insulin. Which approach is best for the nurse to use to evaluate do you effectiveness of the teaching? a) Ask the adolescent to describe his level of comfort with injecting himself with insulin. b) Observe him as he demonstrates self-injection technique in another diabetic adolescent c) Have the adolescent list the procedural steps for safe insulin administration. d) Review his glycosylated hemoglobin level 3 months after the teaching session. Rational: watching the adolescent perform the procedure with another adolescent provides peer support the most information regarding his skill with self-injection. Other options do not provide information about the effectiveness of nurse’s teaching. 131. Nurses working on a surgical unit are concerned about the physicians treatment of clients during invasive procedures, such as dressing changes and insertion of IV lines. Clients are often crying during the procedures, and the physician is usually unconcerned or annoyed by the client‟s response. To resolve this problem, what actions should the nurses take? (Arrange from the first action on the top of the list on the bottom) a) Talk to the physician as a group in a non-confrontational manner. b) Document concerns and report them to the charge nurse. c) Submit a written report to the director of nursing. d) Contact the hospital‟s chief of medical services. e) File a formal complaint with the state medical board. Rational: nurses have both an ethical and legal responsibility to advocate for clients’ physical and emotional safety. Talking with the physician in a non-confrontational manner is the first step in conflict resolution. If this is not effective, the organizational chain of ineffective, a formal complaint with the state medical board should be implemented. 132. When teaching a group of school-age children how to reduce the risk of Lyme disease which instruction should the camp nurse include? a) Wash hands frequently b) Avoid drinking lake water c) Wear long sleeves and pants d) Do not share personal products Rationale: Lyme disease is it tick bone disorder and is transmitted to a child via a tick bite. Keeping the skin covered reduces the risk of being bitten by a tick. Other options are not reduce the risk for tick bites.133. A client with myasthenia Gravis (MG) is receiving immunosuppressive therapy. Review recent laboratory test results show that the client‟s serum magnesium level has decreased below the normal range. In addition to contacting the healthcare provider, what nursing action is most important? a) Check the visual difficulties b) Note most recent hemoglobin level c) Assessed for he and Hand joint pain d) Observe rhythm on telemetry monitor Rationale: If not treated a low little Serum magnesium level can affect myocardial depolarization leading to a lethal arrhythmia, and the nurse should assess for dysrhythmias before contacting the healthcare provider. Other choices are common in MG but do not contribute the Safety risk of low magnesium levels. 134. The nurse is assessing a postpartum client who is 36 hours post-delivery. Which finding should the nurse report to the healthcare provider? a) White blood count of 19,000 mm3 b) Oral temperature of 100.6 F c) Fundus deviated to the right side d) Breasts are firm when palpated Rationale: A temperature greater than 100.4 F (38 C) (B), which is indicative of endometriosis (infection of the lining of the uterus), should be reported to the health care provider. (A and D) are findings that are within normal limits in the postpartum period. Fundal deviation to one side (C) is an expected finding related to a full bladder, so the nurse should encourage the client to void. 135. The nurse asks the parent to stay during the examination of a male toddler‟s genital area. Which intervention should the nurse implement? a) Examine the genitalia as the last part of the total exam. b) Use soothing statements to facilitate cooperation c) Allow the child to keep underpants on to examine genitalia d) Work slowly and methodically so not to stress the child 136. An Insulin infusion for a client with diabetes mellitus who is experiencing hyperglycemic hyperosmolar in addition to the client‟s glucose, which laboratory value is most important for the nurse to monitor? a) Serum potassium b) Urine ketones c) Urine albumin d) Serum protein 137. The nurse administers an oral antiviral to a client with shingles. Which finding is most important for the nurse report to the health care provider? a) Decreased white blood cell count b) Pruritus and muscle aches c) Elevated liver function tests d) Vomiting and diarrhea Rationale: Elevated liver function enzymes are a serious side effect of antivirals and should be reported. A decrease white blood count is a consistent finding with shingle B and (C and D) are side effects that affect that are of less priority than A. 138. A client in the intensive care unit is being mechanically ventilated, has an indwelling urinary catheter in place, and is exhibiting signs of restlessness. Which action should the nurse take first? a) Review the heart rhythm on cardiac monitors b) Check urinary catheter for obstruction c) Auscultated bilateral breath sounds d) Give PRN dose of lorazepam (Ativan) Rationale: Restlessness often results from decreased oxygenation so breath sounds should be assessed first. Giving an anxiolytic such as lorazepam, might be indicated but first the client should be assessed for the cause of the restlessness. An obstruction in the urinary drainage system can cause a distended bladder that may result in restlessness, but patent airway is the priority intervention. The client should be assessed before evaluating the cardiac rhythm on the monitor. 139. Which medication should the nurse anticipate administering to a client who is diagnosed with myxedema coma? a) Intravenous administration of thyroid hormones b) Oral administration of hypnotic agents c) Intravenous bolus of hydrocortisone d) Subcutaneous administration of vitamin k Rationale: The high mortality of myxedema coma requires immediate administration of IV thyroid hormones (A). (B) Is contraindicated, because eves small doses can cause profound somnolence lasting longer than expected. (C) Is administered to clients diagnosed with adrenal insufficiency (Addisonian crisis) and (D) to clients who have had an overdose of warfarin. 140. The first paddle has been placed on the chest of a client who needs defibrillation. Where should the nurse place the second paddle? (Mark the location where the second paddle should be placed on the image). 141. The nurse reviews the signs of hypoglycemia with the parents of a child with Type I diabetes mellitus. The parents correctly understand signs of hypoglycemia if they include which symptoms? a) Fruity breath odorb) Polyphagia c) Diaphoresis d) Polydipsia 142. A client with angina pectoris is being discharge from the hospital. What instruction should the nurse plan to include in this discharge teaching? a) Engage in physical exercise immediately after eating to help decrease cholesterol levels. b) Walk briskly in cold weather to increase cardiac output c) Keep nitroglycerin in a light-colored plastic bottle and readily available. d) Avoid all isometric exercises, but walk regularly. Rationale: Isometric exercise can raise blood pressure for the duration of the exercise, which may be dangerous for a client with cardiovascular disease, while walking provides aerobic conditioning that improves ling, blood vessel, and muscle function. Client with angina should refrain from physical exercise for 2 hours after meals, but exercising does not decrease cholesterol levels. Cold water cause vasoconstriction that may cause chest pain. Nitroglycerin should be readily available and stored in a dark-colored glass bottle not C, to ensure freshness of the medication. 143. The mother of the 12- month-old with cystic fibrosis reports that her child is experiencing increasing congestion despite the use of chest physical therapy (CPT) twice a day and has also experiences a loss of appetite. What instruction should the nurse provide? a) Perform CPT after meals to increase appetite and improve food intake. b) CPT should be performed more frequently, but at least an hour before meals. c) Stop using CPT during the daytime until the child has regained an appetite. d) Perform CPT only in the morning, but increase frequency when appetite improves. Rationale: CPY with inhalation therapy should be performed several times a day to loosen the secretions and move them from the peripheral airway into the central airways where they can be expectorated. CPT should be done at least one hour before meals or two hours after meals. 144. While assessing a client‟s chest tube (CT), the nurse discovers bubbling in the water seal chamber of the chest tube collection device. The client‟s vital signs are: blood pressure of 80/40 mmHg, heart rate 120 beats/minutes, respiratory rate 32 breaths/minutes, oxygen saturation 88%. Which interventions should the nurse implement? a) Provide supplemental oxygen b) Auscultate bilateral lung fields c) Administer a nebulizer treatment d) Reinforce occlusive CT dressing e) Give PRN dose of pain medication Rationale: the air bubbles indicate an air leak from the lungs, the chest tube site, or the chest tube collection system. Providing oxygen improves the oxygen saturation until the leak has been resolved. Auscultating the lung fields helps to identify absent or decrease lung sound due to collapsing lung. 145. The nurse is planning care for a client who admits having suicidal thoughts. Which client behavior indicates the highest risk for the client acting on these suicidal thoughts? a) Express feelings of sadness and loneliness b) Neglects personal hygiene and has no appetitec) Lacks interest in the activity of the family and friends d) Begin to show signs of improvement in affect Rationale: when a depressed client begins to show signs of improvement, it can be because the client has "figured out" how to be successful in committing suicide. Depressed clients, particularly those who have shown signs of potentially becoming suicidal, should be watched with care for an impending suicide attempt might be greater when the client appear suddenly happy, begin to give away possessions, or becomes more relaxed and talkative. 146. The RN is assigned to care for four surgical clients. After receiving report, which client should the nurse see first? The client who is a) Two days postoperative bladder surgery with continuous bladder irrigation infusing. b) One day postoperative laparoscopic cholecystectomy requesting pain medication. c) Three days postoperative colon resection receiving transfusion of packed RBCs. d) Preoperative, in buck‟s traction, and scheduled for hip arthroplasty within the next 12 hours. 147. The nurse ends the assessment of a client by performing a mental status exam. Which statement correctly describes the purpose of the mental status exam? a) Determine the client‟s level of emotional functioning‟ b) Assess functional ability of the primary support system. c) Evaluate the client’s mood, cognition and orientation. d) Review the client‟s pattern of adaptive coping skill Rational: the mental status exam assesses the client for abnormalities in cognitive functioning; potential thought processes, mood and reasoning, the other options listed are all components of the client’s psychosocial assessment. 148. When planning care for a client with acute pancreatitis, which nursing intervention has the highest priority? a) Withhold food and fluid intake. b) Initiate IV fluid replacement. c) Administer antiemetic as needed. d) Evaluate intake and output ratio. Rational: The pathophysiologic processes in acute pancreatitis result from oral fluid and ingestion that causes secretion of pancreatic enzymes, which destroy ductal tissue and pancreatic cells, resulting in auto digestion and fibrosis of the pancreas. The main focus of the nursing care is reducing pain caused by pancreatic destruction through interventions that decrease GI activity, such as keeping the client NPO. Other choices are also important intervention but are secondary to pain management. 149. An adult client with severe depression was admitted to the psychiatric unit yesterday evening. Although the client ran one year ago, his spouse states that the client no longer runs, bur sits and watches television most of the day. Which is most important for the nurse to include in this client‟s plan of care for today? a) Assist client in identifying goals for the day. b) Encourage client to participate for one hour in a team sport. c) Schedule client for a group that focuses on self-esteem.d) Help client to develop a list of daily affirmations. Rationale: clients with severe depression have low energy and benefit from structured activities because concentration is decreased. The client participate in care by identifying goals for the day is the most important intervention for the client’s first day at the unit. Other options can be implemented over time, as the depression decreases. 150. The nurse instructs an unlicensed assistive personnel (UAP) to turn an immobilized elderly client with an indwelling urinary catheter every two hour. What additional action should the nurse instruct the UAP to take each time the client is turned? a) Empty the urinary drainage bag b) Feed the client a snack c) Offer the client oral fluids d) Assess the breath sounds Rationale: Increasing oral fluid intake reduces the risk of problems associated with immobility, so the UAP should be instructed to offer the client oral fluids every two hours, or whenever turning he client. It is not necessary to empty the urinary bag or feed the client every two hours. Assessment is a nursing function, and UAPs do not have the expertise to perform assessment of breath sounds. 151. A female client who was mechanically ventilated for 7 days is extubated. Two hours later…productive cough, and her respirations are rapids and shallow. Which intervention is most important? a) Review record of recent analgesia b) Provide frequent pulmonary toilet c) Prepare the client for intubation d) Obtain STAT arterial blood gases e) 152. The nurse is assessing a first day postpartum client. Which finding is most indicative of a postpartum infection? Moderate amount of foul-smelling lochia. 153. The home health nurse is assessing a male client who has started peritoneal dialysis (PD) 5 days ago. Which assessment finding warrants immediate intervention by the nurse? a) Finger stick blood glucose 120 mg/dL post exchange b) Arteriovenous (AV) graft surgical site pulsations. c) Anorexia and poor intake of adequate dietary protein d) Cloudy dialysate output and rebound abdominal pain 154. In determine the client position for insertion of an indwelling urinary catheter, it is most important for the nurse to recognize which client condition? a) High urinary PH b) Abdominal Ascites c) Orthopnea d) Fever. Rationale: If the client is orthopneic, the nurse needs to adapt the insertion position that does not place the client in a supine position (the head of the bed should be elevated as much as possible).155. The husband of an older woman, diagnosed with pernicious anemia, calls the clinic to report that his wife still has memory loss and some confusion since she received the first dose of nasal cyanocobalamin two days ago. He tells the nurse that he is worried that she might be getting Alzheimer‟s disease. What action should the nurse take? Explain that memory loss and confusion are common with vitamin B12 deficiency 156. An IV antibiotic is prescribed for a client with a postoperative infection. The medication is to be administered in 4 divided doses. What schedule is best for administering this prescription? 1000, 1600, 2200, 0400 157. A client with Addison‟s disease becomes weak, confused, and dehydrated following the onset of an acute viral infection. The client‟s laboratory values include; sodium 129 mEq/l (129mmol/l SI), glucose 54 mg/dl (2.97mmol/l SI) and potassium 5.3 mmol/l SI). When reporting the findings to the HCP, the nurse anticipates a prescription for which intravenous medications? A. Regular insulin. B. Hydrocortisone C. Broad spectrum antibiotic D. Potassium chloride 158. A young adult female with chronic kidney disease (CKD) due to recurring pyelonephritis is hospitalized with basilar crackles and peripheral edema. She is complaining of severe nausea and the cardiac monitor indicates sinus tachycardia with frequent premature ventricular contraction. Her blood pressure is 200 /110 mm Hg, and her temperature is 101 F which PRN medication should the nurse administers first? A. Enalapril B. Furosemide C. Acetaminophen D. Promethazine 159. When entering a client‟s room to administer an 0900 IV antibiotic, the nurse finds that the client is engaged in sexual activity with a visitor. Which actions should the nurse implement? A. Ignore the behavior and hang the IV antibiotic B. tell the client to stop the inappropriate behavior C. Leave the room and close the door quietly D. Complete an unusual occurrence report 160. A child with heart failure (HF) is taking digitalis. Which signs indicates to the nurse that the child may be experiencing digitalis toxicity? a) Tachycarcia b) Dyspnea c) Vomiting d) Muscle cramps161. The nurse applies a blood pressure cuff around a client‟s left thigh. To measure the client‟s blood pressure, where should the diaphragm of the stethoscope be placed? (Mark the location on one of the images.) “On left thigh with arrow pointing to inner thigh” 162. The nurse is preparing to administer an IV dose of ciprofloxacin to a client with urinary tract infection. Which client data requires the most immediate intervention by the nurse? A. Urine culture positive for MRSA B. Serum sodium of 145 mEq/L (145 mmol/L SI) C. Serum creatinine of 4.5 mg/dl (398 mcmol/L SI) D. White blood cell count of of 12,000 mm3 (12 x 109 /L SI) 163. A client with multiple sclerosis is receiving beta-1b interferon every other day. To assess for possible bone marrow suppression caused by the medication, which serum laboratory test findings should the nurse monitor? (Select all that apply) A. Platelet count B. Red blood cell count (RBC) C. White blood cell count (WBC). D. Albumin and protein E. Sodium and potassium 164. A 17-year –old male is brought to the emergency department by his parents because he has been coughing and running a fever with flu-like symptoms for the past 24 hours. Which intervention should the nurse implement first? a) Obtain a chest X-ray per protocol. b) Place a mask on the client’s face. c) C. Assess the client‟s temperature. d) Determine the client‟s blood pressure 165. A client is admitted to a mental health unit after attempting suicide by taking a handful of medications. In developing a plan of care for this client, which goal has the highest priority? a) Signs a no-self-harm contract. b) B. Sleep at least 6 hours nightly. c) C. Attends group therapy every day d) Verbalizes a positive self-image. 166. The charge nurse is making assignments for one practical nurse (PN) and three registered nurses (RN) who are caring for neurologically compromised clients. which client with which change in status is best to assign to the PN? a) Diabetic ketoacidosis whose Glasgow Coma Scale score changed from 10 to 7 b) Subdural hematoma whose blood pressure changed from 150/80 mmHg to 170/60 mmHg c) Myxedema coma whose blood pressure changed from 80/50 mmHg to 70/40 mmHg d) Viral meningitis whose temperature changed from 101° F ( 38.3 C) to 102° F (38.9C).167. A client is admitted to the intensive care unit with diabetes insipidus due to a pituitary gland tumor. Which potential complication should the nurse monitor closely? a- Hypokalemia b- Ketonuria. c- Peripheral edema d- Elevated blood pressure 168. A nurse working on an endocrine unit should see which client first? a) An adolescent male with diabetes who is arguing about his insulin dose. b) An older client with Addison‟s disease whose current blood sugar level is 62mg/dl (3.44 mmol/l). c) An adult with a blood sugar of 384mg/dl (21.31mmol/l) and urine output of 350 ml in the last hour. d) A client taking corticosteroids who has become disoriented in the last two hours. 169. The nurse is caring for a group of a clients on a surgical unit. Which client should the nurse assess first? a) A client who is two days post knee surgery and who describes pain at “4” on a 1 to 10 scale b) A client who is one day post bowel resection with no bowel sounds c) A client who is 8 hours post appendectomy with urinary output of 480 ml d) A client who was admitted with severe abdominal pain and suddenly has no pain 170. A client with diabetes mellitus tells the nurse that she uses cranberry juice to help prevent urinary tract infection. What instruction should the nurse provide? a) Be sure to drink sugar-free cranberry juice b) Drinking cranberry juice does not prevent infection c) Cranberries may increase the effect of insulin d) Excess cranberry juice can be constipating 171. What intervention should the nurse implement during the administration of a vesicant chemotherapeutic agent via an IV site in the client's arm? a) Assess IV site frequently for signs of extravasation. b) Monitor capillary refill distal to the infusion site c) Apply a topical anesthetic at the infusion site for burning d) Explain that temporary burning at the IV site may occur 172. When obtaining subjective data from a client, what intervention should the nurse implement first? a) listen attentively b) establish rapport c) list client problems d) clarify inferences 173. When implementing a disaster intervention plan, which intervention should the nurse implement first? a) Initiate the discharge of stable clients from hospital unitsb) Identify a command center where activities are coordinated c) Assess community safety needs impacted by the disaster d) Instruct all essential off-duty personnel to report to the facility. 174. Four clients arrive on the labor and delivery unit at the same time. Which client should the nurse assess first? a) A 38-week primagravida who reports contractions occurring every 10 minutes b) A 39-week primagravida with biophysical profile score of 5 out of 8. c) A 41-week multigrada who is scheduled induction of labor today d) A 36-week multigrada with a prescription for serial blood pressure 175. When preparing a client for discharge from the hospital following a cystectomy and urinary diversion to treat bladder cancer, which instruction is most important for the nurse to include in the client´s discharge teaching plan? A- Report any signs of cloudy urine output. B- Seek counseling for body image concerns. C- Frequently empty bladder to avoid distension. D- Follow instructions for self-care toileting 176. The nurse is preparing to send a client cardiac catheterization lab for an angioplasty. Which client report is most important for the nurse to explore further prior to start of the procedure? a) Verbalizes a fear of being in a confined space. b) Drank a glass of water in the past 2 hours. c) Reports left chest wall pain prior to admission. d) Experiences facial swelling after eating crab 177. A male client approaches the nurse with an angry expression on his face and raises his voice, saying, “My roommate is the most selfish, self –centered, angry person I have ever met. If the loses his temper one more time with me, I am going to punch him out‟‟ The nurse recognizes that the client is using which defense mechanism? a) Denial b) Splitting c) Projection d) Rationalization 178. Which instruction is most important for the nurse to provide a client who receives a new prescription for risedronate sodium to treat osteoporosis a- Remain upright after taking the medication b- Increase intake of foods rich in calcium c- Begin a weight-bearing exercise plan d- Schedule a bone density test every year 179. A client is admitted with hepatitis A (HAV) and dehydration. Subjective symptoms include anorexia, fatigue, and malaise. What additional assessment should the nurse expect to find during the preicteric phase? a- RUQ abdominal pain b- Clay – colored stool c- Icteric sclera d- Pruritis 180. An older male client arrives at the clinic complaining that his bladder always feels full. He complains of a weak urine flow frequent dribbling after voiding and increasing nocturia with difficulty initiating his urine stream action should the nurse implement? a) Palpate the client’s suprapubic area for distention b) Advise the client to maintain a voiding diary for one week c) Instruct in effective techniques to cleanse the glans penis d) Obtain urine specimen for culture and sensitivity 181. A client with pneumonia has arterial blood gases levels at: pH 7.33; PaC02 49 mm/ Hg; HC03 25mEq/L; PaO2 95. What intervention should the nurse implement based on these results? a) Instruct the client to breathe into a paper bag b) Initiate oxygen administration at 2 to 3/L per nasal cannula c) Institute coughing and deep breathing protocols d) Prepare to administer sodium chloride fluids 182. What is the primary purpose for initiating nursing intervention that promote good nutrition, rest, and exercise, and stress reduction for clients diagnosed with an HIV infection? 1. Prevent spread of infection to others 2. Improve function of the immune system 3. Increase ability to carry out activities of daily living 4. Promote a feeling of general well-being 183. After diagnosis and initial treatment of a 3-year-old with Cystic fibrosis, the nurse provides home care instructions to the mother, which statement by the child's mother indicates that she understands home care treatment to promote pulmonary functions? a- Chest physiotherapy should be performed twice a day before a meal b- Energy should be conserved by scheduling minimally strenuous c- Administer a cough suppressant every 8 hour d- Maintain supplemental oxygen at 4 to 6 L/minute 184. Which laboratory results should the nurse closely monitor in a client who has end – stage renal disease (ESRD)  Serum Potassium, calcium, and phosphorus  Erythrocytes, hemoglobin, and hematocrit  Blood pressure, heart rate and temperature  Leukocytes, neutrophils, and thyroxine 185. A client who is admitted to the medical unit with fluid volume overload has 2+ dependent edema in the lower extremities and a serum sodium level of 115mEq/L. Which assessment should the nurse complete next?a) Skin turgor. b) Mental status. c) Bowel sound. d) Urinary output. 203- A male client with hypercholesterolemia wants to change his diet to help reduce his cholesterol levels. When breakfast items should the nurse encourage the client to eat? (Select all that apply) a- Sausage patties and eggs b- Whole wheat toast and jam c- Bagels and cream cheese d- Toaster pastries and milk e- Blackberries and oatmeal 204- After an unsuccessful resuscitation attempt, the nurse calls the family of the deceased. The family wish to see the body before it is taken to the funeral home. Which interventions should the nurse take to prepare the body before the family enters the room? (Select all that apply) a- Take out dentures and place in a labeled cup b- Apply a body shroud c- Place a small pillow under the head d- Remove resuscitation equipment from the room e- Gently close the eyes 205- A client with major depression who is taking fluoxetine calls the psychiatric clinic reporting being more agitated, irritable, and anxious than usual. Which intervention should the nurse implement? a- Tell the client to have a complete blood count (CBC) drawn b- Instruct the client to seek medical attention immediately c- Encourage him to take the medication at night with a snack d- Explain that these are common side effects of the medication 206- A client with diabetic peripheral neuropathy has been taking pregabalin for 4 days. Which finding indicates to the nurse that the medication is effective? a- Granulating tissue in foot ulcer b- Full volume of pedal pulse c- Reduced level of pain d- Improved visual activity. 207- The nurse is assessing an older adult with type 2 diabetes. Which assessment finding indicates that the client understands long- term control of diabetes? a- The fating blood sugar was 120 mg/dl this morning. b- Urine ketones have been negative for the past 6 months c- The hemoglobin A1C was 6.5g/100 ml last week d- No diabetic ketoacidosis has occurred in 6 months.208- The nurse is assessing a client with Addison's disease who is weak, dizzy, disoriented, and has dry oral mucous membranes, poor skin turgor, and sunken eyes. Vital signs are blood pressure 94/44, heart rate 123 beats/minute, respiration 22 breaths/minute. Which intervention should the nurse implement first? a- Assess extremity strength and resistance b- Report a sodium level of 132 mEq/L or mmol/L (SI units) c- Measure and record the cardiac QRS complex d- Check current finger stick glucose 209- The nurse assesses an older adult who is newly admitted to a long-term care facility. The client has dry, flaky skin and long thickened fingernails. The clients have a medical history of a stroke which resulted in left-sided paralysis and dysphagia. In planning care for the client, which task should the nurse delegate to the unlicensed personnel (UAP)? a- Soak and file fingernails b- Offer fluids frequently c- Monitor skin elasticity d- Ambulate in the hallway 210- A client is receiving lidocaine IV at 3 mg/minute. The pharmacy dispenses a 500 ml IV solution of normal saline (NS) with 2 grams of lidocaine. The nurse should regulate the infusion pump to deliver how many ml/hr? (Enter numeric value only. If rounding to the nearest whole number.) 45 211- The nurse is demonstrating wound care to a client following abdominal surgery. In what order should the nurse teach the technique? (Arrange from first action on top to last action on bottom) Remove old dressing using clean gloves. Discard gloves with old dressing Moisten sterile gauze with normal saline. Clean wound from least contaminated area to most contaminated area” Apply sterile gauze dressing to wound area Secure dressing with tape 212- The healthcare provider explains through an interpreter the risks and benefits of a scheduled surgical procedure to a non-English speaking female client. The client gives verbal consent and the healthcare provider leaves, instructing the nurse to witness the signature on the consent form. The client and interpreter then speaker together in the foreign language for an additional 2 minutes until the interpreter concludes, “She says it is OK.” What action should the nurse take next?a- Clarify the client‟s consent through the use of gesture and simple terms b- Have the interpreter co-sign the consent to validate client understanding c- Ask for full explanation from the interpreter of the witnessed discussion d- Have the client sign the consent and the nurse witness the signature 214- After receiving report, the nurse can most safely plan to assess with client last? a- An older client with dark red drainage on a postoperative dressing, but no drainage in the Hemovac b- An adult client with no postoperative drainage in the Jackson- Pratt drain with the bulb compressed c- An older client with a distended abdomen and no drainage from the nasogastric tube d- An adult client with rectal tube draining clear pale red liquid drainage 215- The nurse is assigned to care for a client diagnosed with psoriasis. What behavior by the nurse addresses this client's psychosocial need for acceptance? a- Wearing gloves when interviewing the client b- Encouraging the client to join a support group c- Shaking the client's hand during an introduction. d- Allowing the client to ventilate feelings 216- A 41-week gestation primigravida woman is admitted to labor and delivery for induction of labor. What finding should the nurse report to the healthcare provider before initiating the infusion of oxytocin? a- Fetal heart tones located in upper right quadrant b- Biophysical profile results showing oligohydramnios c- Regular contractions occurring every 10 minutes d- Sterile vaginal exam reveling 3 cm dilatation 218- After learning that she has terminal pancreatic cancer, a female client becomes very angry and says to the nurse, “God has abandoned me. What did I do to deserve this?” Based on this response, the nurse decides to include which nursing problem in the client‟s plan of care? a- Ineffective coping b- Spiritual distress c- Acute pain d- Complicated grieving 219- Un infant is unresponsive and gasping for breath. Prior to starting CPR, which site should the nurse palpate for a pulse? Brachial pulse Infant <1 year old, brachial pulse Children 1 year to puberty, carotid or femoral 220- A group of nurses implement a pilot study to evaluate a proposed evidence-based change to providing client care. Evaluation indicates successful outcomes and the nurses want to integrate the change throughout the facility. Which action should be taken? (Select all that apply) a- Invite data review by the quality improvement department b- Submit a sentinel event report to the research committeec- Propose clinical practice guidelines to the nursing committee d-Obtain informed consent from clients who will receive care e-Arrange inservice training through the educational department 221- The mother of a school age child calls the school to ask when her daughter can return to school after treatment for Pediculosis capitis. What is the nurse best response? (nits liendra) a- When the classroom epidemic subsides b- Two weeks after the last treatment c- As soon as the itching stops d- After the treatment kills all the live lice 222- When assessing a client with acute asthma, the nurse is most likely to obtain which finding? a- Pursed lip breathing and clubbing of fingers b- Fever and a high- pitched inspiratory stridor c- A short expiratory phase and hemoptysis d- Cough and musical breath sound on expiration 223- A client with multiple sclerosis is experiencing scotomas (blind spots), which are limiting peripheral vision. What intervention should the nurse include in this client's plan of care? a- Encourage the use of corrective lenses during the day b- Practice visual exercises that focus on a still object c- Alternate an eye patch from eye every 2 hours d- Teach techniques for scanning the environment. 225- Which intervention should the nurse include in the plan of care for a patient with tetanus? Open window shades to provide natural light a- Encourage coughing and deep breathing b- Minimize the amount of stimuli in the room c- Reposition from side to side every hour 226- A client who is newly diagnosed with type 2 diabetes mellitus (DM) receives a prescription for metformin (Glucophage) 500 mg PO twice daily. What information should the nurse include in this client‟s teaching plan? (Select all that apply.) a- Take an additional dose for signs of hyperglycemia b- Recognize signs and symptoms of hypoglycemia. c- Report persist polyuria to the healthcare provider. d- Use sliding scale insulin for finger stick glucose elevation. e- Take Glucophage with the morning and evening meal. 227- A client collapses while showering and is found discovered by the nurse while making rounds. The client is not breathing and does not have a palpable pulse. The nurse obtains the Automated External Defibrillator (AED). What action should the nurse implement next? a- Follow the prompts of the AED b- Apply the AED pads to the client‟s chest c- Wipe the client‟s chest dry d- Move the client from the bathroom228- A female client with cancer tells the home care nurse that she has a good appetite but experiences nausea whenever she smells food cooking. What action should the nurse implement? a- Encourage family members to cook meals outdoors and bring the cooked food inside b- Advise the client to replace cooked foods with a variety of different nutritional supplements c- Assess the client‟s mucus membranes and report the findings to the healthcare provider d- Instruct the clients to take an antiemetic before every meal to prevent excessive vomiting. 229- At 1130, the nurse assumes care of an adult client with diabetes mellitus who was admitted with an infected foot ulcer. After reviewing the client‟s electronic health record, which priority nursing action should the nurse implement? Click on each chart tab for additional information. Please be sure to scroll to the bottom right corner of each tab to view all information contained in the client‟s medical record. a- Administer insulin per sliding scale b- Assess appearance of foot wound c- Obtain antibiotic peak and trough levels d- Initiate hourly urine output measurements 230- Following morning care, a client with C-5 spinal cord injury who is sitting in a wheelchair becomes flushed and complains of a headache. Which intervention should the nurse implement first? a-Relieve any kinks or obstruction in the client’s Foley tubing b-Asses the client‟s blood pressures every 15 minutes c-Administer a prescribed PRN dose of hydralazine (Apresoline) e-Teach the client to recognize symptoms of dyreflexia 231- After a motor vehicle collision a client admitted to the medical unit with acute adrenal insufficiency (Addisonian crisis). Which prescription should the nurse implement? a- Determine serum glucose levels b- Withhold potassium additives to IV fluids c- Give IV corticosteroid replacement d- Prepare to initiate IV vasopressors232- The unlicensed assistive personnel (UAP) reports that a client‟s blood pressure cannot be measured because the client has casts on both arms and is unable to be turned to the prone position for blood pressure measurement in the legs. What action should the nurse implement? a- Advise the UAP to document the last blood pressure obtained on the client graphic sheet b- Estimate the blood pressure by assessing the pulse volume of the client‟s radial pulses c- Demonstrate how to palpate the popliteal pulse with the client supine and the knee flexed d- Document why the blood pressure cannot be accurately measured at the present time 233- The nurse identifies the presence of clear fluid on the surgical dressing of a client who just returned to the unit following lumbar spinal surgery. What action should the nurse implement immediately? a- Change the dressing using a compression bandage b- Test fluid on the dressing for glucose c- Document the findings in the electronic medical record d- Mark drainage area with a pen and continue monitor 234- A male client with diabetes mellitus takes NPH/ regular 70/30 insulin before meals and azithromycin PO daily, using medication he brought from home. When the nurse delivers his breakfast tray, the client tells the nurse that he took his insulin but forgot to take his daily dose of the azithromycin an hour before breakfast as instructed. What action should the nurse implement? a- Provide a PRN dose of an antacid to take with the azithromycin right after breakfast b- Offer to obtain a new breakfast tray in an hour so the client can take the azithromycin c- Instruct the client to eat his breakfast and take the azithromycin two hours after eating d- Tell the client to skip that day‟s dose and resume taking the azithromycin the next day 235- After administering a proton pump inhibitor (PPI), which action should the nurse take to evaluate the effectiveness of the medication? a- Ask the client about gastrointestinal pain b- Measure the client‟s fluid intake and output c- Monitor the client‟s serum electrolyte levels d- Auscultate for bowel sounds in all quadrants 238- A female client on the mental health unit frequently asks the nurse when she can be discharged. Then, becoming more anxious, she begins to pace the hallway. What intervention should the nurse implement first? a- Review the current treatment plan with the client b- Inform the healthcare provider about the client‟s behaviors c- Determine if the client has PRN medication for anxiety d- Explore the client’s reasons for wanting to be discharged.239- The nurse working on a mental health unit is prioritizing nursing care activities because of a staffing shortage. One practical nurse (PN) is on the unit with the nurse, and another RN is expected to arrive within two hours. Clients need to be awakened and morning medications need to be prepared. Which plan is best for the nurse to implement? a- Wake all the clients and instruct them go to dining area for medication administration b- Explain to the clients that it will be necessary to cooperate until another RN arrives c- Ask the PN to administer medications as clients are awakened so both nurses are available d- Allow the clients to sleep until a third staff person can assist with unit activities 240- A 5-week-old infant who developed projectile vomiting over the last two weeks is diagnosed with hypertrophic pyloric stenosis. Which intervention should the nurse plan to implement? a- Instruct the mother to give the child sugar water only b- Maintain intravenous fluid therapy per prescription c- Provide Pedialyte feedings via the nasogastric tube d- Offer the infant Pedialyte feedings every 2 hours. 241- A male client with hypertension, who received new antihypertensive prescriptions at his last visit returns to the clinic two weeks later to evaluate his blood pressure (BP). His BP is 158/106 mmHg and he admits that he has not been taking the prescribed medication because the drugs make him “feel bad”. In explaining the need for hypertension control, the nurse should stress that an elevated BP places the client at risk for which pathophysiological condition? a- Stroke secondary to hemorrhage b- Acute kidney injury due to glomerular damage c- Heart block due to myocardial damage d- Blindness secondary to cataracts 242- A woman at 24-weeks gestation who has fever, body aches, and has been coughing for the las 5 days is sent to the hospital with admission prescriptions for H1N1 influenza. Which prescriptions has the highest priority? a- Ringers Lactate IV 125ml/8 hours b- Obtain specimens for cultures c- Assign private room d- Vital signs q4 hours 243- A client with deep vein thrombosis (DVT) in the left leg is on a heparin protocol. Which intervention is most important for the nurse to include in this client's plan of care? a- Observe for bleeding side effects related to heparin therapy. b- Assess blood pressure and heart rate at least q4 hours c- Measure calf girths to evaluate edema in the affected leg d- Encourage mobilization to prevent pulmonary embolism 245- A preschool teacher notifies the school nurse that child A has bitten child B on the arm. Child B‟s skin is broken, but is not bleeding. What action should the school nurse take first? a- Apply antibiotic cream to Child B‟s arm immediately b- Determine if Child A has a history of Hepatic C or HIV c- Determine the date of Child B‟s latest tetanus boosterd- Wash Child B’s arm thoroughly with soap and water 246- At a community health fair, a 50-year-old woman tells the nurse that she has an annual physical exam that includes a clinical breast exam and an annual mammogram. How should the nurse respond? a- Ask the woman if she also performs monthly breast self-exams. b- Advise the woman that mammograms are only needed every two years at her age. c- Encourage the woman to explore her fears about breast cancer. d- Comment the woman for adhering to the recommended cancer detection guidelines. 247- Prior to surgery, written consent must be obtained. What is the nurse‟s legal responsibility with regard to obtaining written consent? a- Validate the clients understanding of the surgical procedure to be conducted b- Explain the surgical procedure to the client ask the client to sign the consent form c- Ask the client or a family member to sign the surgical consent form d- Determine that the surgical consent form has been signed and is included in the client’s record 248- The nurse provides sliding scale insulin administration instruction to an adult who was recently diagnose with diabetes mellitus. The client demonstrates and understanding on the instructions provided by performing the procedure in which order? (Arrange with the first on top and the last on the bottom.) Obtain blood glucose level Verify the insulin prescription Draw insulin into insulin syringe Cleanse the selected site 249- What is the primary focus of postoperative nursing care for the client with colon trauma? a- Monitoring for elevated coagulation studies b- Observation for and prevention of fistulas c- Monitoring for signs of hyponatremia d- Observation for and prevention of infection 250- While assessing a client four hours post-thoracentesis, the nurse is unable to auscultate breath sounds on the right side of the chest. What action should the nurse take first? a- Instruct the client to perform cough and deep breathing exercises b- Assess the client’s vital signs and respiratory effort c- Administer oxygen per nasal canula per PNR protocold- Document assessment findings in client‟s medical record 251- A client who is mechanically ventilated is receiving continuous enteral feedings through a nasogastric feeding tube. To prevent aspiration, which intervention is most important for the nurse to implement? a- Verify the feeding tube position with a daily chest x-ray b- Maintain head of bed elevated while enteral feeding is infusing c- Check feeding tube placement with air bolus prior to use d- Aspirate stomach contents every 4 hours to assess residuals 252- After years of struggling with weight management, a middle-age man is evaluated for gastroplasty. He has experienced difficulty with managing his diabetes mellitus and hypertension, but he is approved for surgery. Which intervention is most important for the nurse to include in this client‟s plan of care? a- Monitor for urinary incontinence b- Apply sequential compression stockings c- Provide a wide variety of meal choices d- Observe for signs of depressions 253- After receiving report on an inpatient acute care unit, which client should the nurse assess first? a- The client with bowel obstruction due to a volvulus who is experiencing abdominal rigidity b- The client who had surgery yesterday and is experiencing a paralytic ileus with absent bowel sounds c- The client with an obstruction of the large intestine who is experiencing abdominal distention d- The client with a small bowel obstruction who has a nasogastric tube that is draining greenish fluid 254- An adolescent male client is admitted to the hospital. Based on Erikson‟s theory of psychosocial development, which nursing intervention best assists this adolescent‟s adjustment to his hospital stay? a- Invite him to participate in the evening group activity b- Schedule frequent private phone calls to his parents c- Provide access to a variety of video games in his room d- Encourage him to learn his way around the hospital 255- A client with skin grafts covering full-thickness burns on both arms and legs is scheduled for a dressing change. The client is nervous and requests that the dressing change be skipped this time. What action is most important for the nurse to take? a- Explain the importance of regular dressing change b- Administer an anti-anxiety medication c- Proceed with the scheduled dressing change d- Encourage the client to express any anxieties256- A continuous infusion of nitroglycerin is prescribed for an adult male admitted with an acute myocardial infarction. The client is experiencing active chest pain that he describes as 8 out of 10. Which intervention is most important for the nurse to implement? a- Administer infusion via an infusion pump b- Obtain current serum potassium level c- Continuously monitor blood pressure d- Teach guided imagery to decreased pain 257- When caring for a client with diabetes insipidus (DI), it is most important for the nurse to include frequent assessment for which conditions in the client‟s plan of care? a- Dry mucous membranes, hypotension b- Decreased appetite, headache c- Nausea and vomiting, muscle weakness d- Elevated blood pressure, petechiae 258- An older male was recently admitted to the rehabilitation unit with unilateral neglect syndrome as the result of a cerebrovascular accident (CVA). Which action should the nurse include in the plan of care? a- Provide additional light in the room to promote sensory stimulation b- Teach the client to turn his head from side to side for visual scanning c- Place a clock and calendar in the room to improve orientation d- Use hand and arm gestures to improve communication and comprehension 259- The nurse is caring for a client with a tracheostomy. Which action should the nurse perform when suctioning the tracheostomy tube? a- Increase wall suction with removal of the suction catheter b- Place sterile saline 10 ml in the tracheostomy tube, then suction as continuous suction is applied c- Suction the client‟s oropharynx before tracheal suctioning d- Insert the suction catheter into the trachea, and apply intermittent suction with removal catheter 260- The urinary drainage of a client with a continuous bladder irrigation is becoming increasingly red. Which intervention should the nurse implement? a- Increase the irrigation rate b- Lower the head of the bed c- Milk the catheter tubing d- Evaluate for fluid overload 261- The nurse is preparing to administer a suspension ampicillin labeled, 250mg/5ml, to a 12-year old child with impetigo. The prescription is for 500 mg QID. How many ml should the child receive per day? (Enter numeric value only) 40 262- A client with a severe prostatic infection that caused a blocked urethra is 3 days post-surgical urinary diversion. The healthcare provider directs the nurse to remove the suprapubic catheter to allow the client to void normally. Which intervention should the nurse implement first?a- Cleanse the site around the catheter b- Use a 20 ml syringe to deflate balloon c- Clamp catheter until a client voids naturally d- Empty urine from urinary drainage bag 263- The nurse receives report on four clients who are complaining of increased pain. Which client requires immediate by the nurse? a- Burning pain due to a Morton‟s neuroma b- Sharp pain related to a crushed femur c- Paresthesia of fingers due to carpal tunnel syndrome d- Stinging pain related to Plantar fasciitis 264- A client morning assessment includes bounding peripheral pulses, weight gain of 2 pounds, pitting ankle edema, and moist crackles bilaterally. Which intervention is most important for nurse to include the in this client‟s plan of care? a- Restrict daily fluid intake to 1500 ml b- Weight client every morning c- Maintain accurate intake and output d- Administer prescribed diuretic 265- A female client who has a borderline personality disorder is being discharged today. When the nurse makes morning rounds, the clients begins the interaction by complaining about the aloofness of the night shift nurse and expresses joy to see that, „‟ My favorite nurse is on duty now” Which response is best for the nurse to provide to this client‟s dichotomous tendency? a- I am glad you like me. Which nurse was acting aloof to you? b- Tomorrow I will talk to that nurse about how you were treated last night c- What did the night nurse that makes you think she is aloof? d- I am happy that you are getting better and will be able to go home 266- The nurse is calculating the one-minute Apgar score for a newborn male infant, and determines that his heart rate is 150 beats/minute, he has a vigorous cry, his muscle tone is good with total flexion, he has quick reflex irritability, and his color is dusky and cyanotic. What Apgar score should the nurse assign to the infant? (Enter the numeric value only.) 8 268- In preparing a nursing care plan for a client admitted with a diagnosis of Guillain-Barre syndrome, which nursing problem has the highest priority? a- Ineffective coping related to uncertainly of disease progression b- Imbalanced nutrition: less than body requirements related to impaired swallowing reflex c- Ineffective breathing pattern related to ascending paralysis. d- Impaired physical mobility related to asymmetrical descending paralysis 269- A newly hired unlicensed assistive personnel (UAP) is assigned to a home healthcare team along with two experienced UAPs. Which intervention should the home health nurse implement to ensure adequate care for all clients? a- Ask the most experienced UAP on the team to partner with the newly hired UAPb- Evaluate the newly hired UAP’s level of competency by observing the UAP deliver care c- Review the UAP‟s skills checklist and experience with the person who hired the UAP d- Assign the newly hired UAP to clients who require the least complex level of care 270- What Nursing intervention is particularly indicated for the second stage of labor? a- Providing pain medication to increase the client‟s tolerance of labor b- Assessing the fetal heart rate and pattern for signs of fetal distress c- Monitoring effects of oxytocin administration to help achieve cervical dilation d- Assisting the client to push effectively so that the expulsion of the fetus can be achieved. 271- An adult male with a 6 cm thoracic aneurysm is being prepared for surgery. The nurse reports to the healthcare provider that the client‟s blood pressure is 220/112 mmHg, so an antihypertensive agent is added to the client‟s IV infusion. Which finding warrants immediate intervention by the nurse? a- Reports a tearing, sharp pain between his shoulder blades b- Blood pressure reading of 200/100 mmHg 15 minutes later c- Rose colored urine draining from the urinary catheter d- Sinus tachycardia with frequent premature ventricular beats (PVC) 272- A client is admitted with the diagnosis of Wernicke‟s syndrome. What assessment finding should the nurse use in planning the client‟s care? a- Right lower abdominal pain b- Confusion c- Depression d- Peripheral neuropathy 273- The nurse is developing a teaching plan for a client with acute gastritis caused by drinking contaminated water. The nurse should emphasize the need to report the onset of which problem? a- Low grade fever b- Bruising of the skin c- Abdominal cramping d- Bloody emesis 274- A client with complaints of shortness of breath and abdominal pain 1 week after bariatric surgery is admitted for follow-up evaluation. Which assessment finding warrants immediate intervention by the nurse? a- Rectal temperature of 101F b- Complaints of left shoulder pain c- Blood pressure of 88/50 mmHg d- Sustained sinus tachycardia 275- A client with prescription for “do not resuscitate” (DNR) begins to manifest signs of impending death. After notifying the family of the client‟s status, what priority action should the nurse implement?a- The client’s need for pain medication should be determined b- The impeding signs of death should be documented c- The nurse manager should be update on the client‟s status d- The client‟s status should be conveyed to the chaplain. 277- A male client with schizophrenia tells the nurse that the hospital has installed cameras that watch him and listening devices that record what everyone says. Which nursing problem should the nurse document for this client? a- Noncompliance with medication related to thought broadcasting b- Situational self esteem disturbance secondary to schizophrenia c- Disturbed sensory perception related to auditory hallucinations d- Impaired environmental interpretation related to paranoid delusions 278- A nurse is administering diazepam, a benzodiazepine, 10 mg IV push PRN, as prescribed to a client with alcohol withdrawal symptoms. Which actions should the nurse implement when administering the medication? (Select all that apply) a- Protect medication from exposure to light b- Monitor for changes in level of consciousness c- Observe for onset of generalized bruising or bleeding d- Perform ongoing assessment of respiratory status e- Administer slowly over at least two minutes 279- An older male client is admitted to the mental health unit with a sudden onset of global disorientation and is continuously conversing with his mother, who died 50 years ago. The nurse reviews the multiple prescriptions the client is currently taking and assesses his urine specimen, which is cloudy, dark yellow, and has a foul odor. These findings suggest that this client is experiencing which condition? a- Psychotic episode b- Dementia c- Delirium d- Depression 280- While changing the pressure ulcer dressing of a client who is immobile, the nurse notes that the boundary edges of the wound have increased. Before reporting this finding to the healthcare provider, the nurse should review which of the client‟s serum laboratory values? a- Potassium b- Platelets c- Creatinine d- Albumin 281- A client is admitted for an exacerbation of heart failure (HF) and is being treated with diuretics for fluid volume excess. In planning nursing care, which interventions should the nurse include? (Select all apply) a- Encourage oral fluid intake of 3,000 ml/day b- Observe for evidence of hypokalemia c- Teach the client how to restrict dietary sodiumd- Monitor PTT, PT, and IRN, lab values e- Weight the client daily, in the morning 282- The nurse provides discharge teaching to a client who was recently diagnosed with diabetes mellitus (DM). After receiving the instructions, the client expresses understanding about when, how, and why to take his prescribed medications at home. Which intervention is most important for the nurse to implement a- Review the purpose of medications prescribed for the client to take home with him b- Provide the client with a printed list of medications and a schedule for administration. c- Send a list of medications taken while hospitalized to the client’s healthcare provider d- Offer to consult with the pharmacist about resources for reduced price medications 283- The nurse is conducting intake interviews of children at a city clinic. Which child is most susceptible to contracting lead poisoning? a- An adolescent who works part time in a pain factory b- A 2- year- old who plays on aging outdoor playground equipment c- A 10 – year – old who has Type 1 diabetes mellitus e- An 8 – year old who lives in a housing project 284- An IV infusing in a client‟s left forearm becomes infiltrated. After removing the IV, which sites should the nurse select as possible site to insert another IV catheter? (Select all that apply) a- Right hand b- Right forearm c- Left hand d- Right subclavian e- Left subclavian 285- A client has had several episodes of clear, watery diarrhea that starter yesterday. What action should the nurse implement? a- Administer a prescribed PRN antiemetic b- Assess the client for the presence of hemorrhoids c- Check the client‟s hemoglobin level d- Review the client’s current list of medications 287- A postpartum client who is bottle feeding develops breast engorgement. What is the best recommendation for the nurse to provide this client? a- Take a prescribed analgesic and exposed breasts to air b- Place warm packs on both of the breasts c- Avoid stimulation of the breasts and wear tight bra d- Express a small amount of breast milk by hand 288- A nurse assesses a client whose hand begins to spasm when the blood pressure cuff is inflated. The client complains of paresthesia in the fingers and toes. Which serum laboratory finding should the nurse expect to find? a- Elevated serum calcium b- Low serum magnesiumc- Low serum calcium d- Elevated serum potassium 289- The parents of a 6-year-old child recently diagnosed with Duchenne muscular dystrophy tell the nurse that their child wants to continue attending swimming classes. How should the nurse respond? a- Encourage the parents to allow the child to continue attending swimming lesson with supervision b- Suggest that the child be encouraged to participate in a team sport to encourage socialization c- Explain that their child is too young to understand the risks associated with swimming d- Provide a list of alternative activities that are less likely to cause the child experience fatigue 290- In performing the admission assessment for a client experiencing complications of long term Parkinson‟s disease, which question by the nurse provides the best information about disease progression? a- Have you experienced any stiffness in your neck or shoulder?” b- “Do you notice any jerky type movements of your arms?” c- “Have you ever been frozen to a spot and unable to move?” d- “Do you have any problems with your hands shaking?” 291- An adult suffered burns to face and chest resulting from a grease fire. On admission, the client was intubated and a 2 liter bolus of normal saline was administered IV. Currently the normal saline is infusing at 250ml/hour. The client‟s heart rate is 120 beats/minute, blood pressure is 90/50 mmHg, respirations are 12 breaths/minute over the ventilated 12 breaths for a total of 24 breaths/minute, and the central venous pressure (CVP)is 4 mm H2O. Which intervention should the nurse implement? a- Increase the oxygen delivered by the ventilator b- Infuse an additional bolus of normal saline c- Lower head of the bed to a recumbent position d- Bring a tracheotomy tray to the bedside 292- The nurse is caring for a client who has silvery scaling plaques bilaterally on elbows, forearms, and palms. When scratched, the skin bleeds over these plaques. What is most important to include in this client‟s plan of care? a- Interventions to decrease emotional stress b- ncrease omega-3 fatty acids in the diet c- Instruct about applying an antifungal ointment 293- After successful resuscitation, a client is given propranolol and transferred to Intensive Coronary Care Unit (ICCU). On admission, magnesium sulfate 4 grams IV in 250 ml D5 W at one gram/hour. Which assessment findings require immediate intervention by the nurse? a- Dark amber urine draining per indwelling catheter with 40 ml per hour b- Serum calcium of 9.0 mg/dl (2.2 mmol/L SI) and magnesium of 1.8 md/dl or Eq/L (0.74 mmol/L SI)c- Sinus rhythm at 72 beats/minute and peripheral blood pressure of 99/62 d- Respiratory rate of 10 breaths per minute and pulse oximetry of 90%. 294- A woman at 12-week gestation comes to the clinic for her first prenatal visit. After completing a health history, the nurse should discuss which topic about pregnancy at this initial visit? a- Complications associated with childbirth b- Concerns about parenting c- Cultural practices related to childbearing d- Knowledge about labor and delivery 295- A client newly diagnosed with diabetes mellitus suddenly becomes confused and weak. Which interventions should the nurse implement? (Select all that apply) a- Give the client 4 ounces of orange juice b- Obtain blood pressure and pulse rate c- Provide the client with ½ cup diet carbonated soda d- Administer a PRN dose of regular insulin e- Check the client’s current finger stick blood glucose 296- The nurse assesses a client who had bilateral total knee replacements (TKR) four hours ago the nurse notes that the dressing on the client‟s right knee is saturated with serosanguineous drainage. What action should the nurse implement? a- Determine if the wound drainage device is functioning correctly b- Confirm that the continuous passive motion device is intact c- Withhold next scheduled dose of low molecular weight heparin d- Monitor the clients current white blood cell count (WBC) 297- An older adult with known cognitive impairment residing in a long- term care facility suddenly becomes disoriented and confused. There are no signs of extremity weakness or other neurological changes. Which intervention should the nurse implement? a- Obtain 12 lead electrocardiogram b- Assess the urine for cloudiness c- Perform stroke assessment d- Auscultate for bowel sounds 298- After receiving the morning report at 0700 on a postpartum unit, which client should the nurse assess first? The client who is a- Complaining of IV site pain whose IV antibiotic is due at 0800 b- 23-hours postpartum and complaining of episiotomy pain. c- A third day post- cesarean requesting assistance to the bathroom d- Requesting assistance with breastfeeding her one-day old infant 299- The nurse observes an unlicensed assistive personnel (UAP) applying an alcohol-based hand rub while leaving a client‟s room after taking vital signs. What action should the nurse take? a- Remind the UAP to continue rubbing the hands together until they dry b- Instruct the UAP to return to the client‟s room to perform handwashingc- Advise the UAP to wear gloves when obtaining vital signs for all clients d- Supervise the UAP in the next client‟s room to evaluate hand hygiene 300- Which client problem has the higher priority for the child sickle cell anemia who has a temperature of 101 F (38.3)? a- Infection related to low platelet count b- Activity intolerance related to anemia c- Fluid volume deficit related to temperature elevation d- Altered urinary elimination related to renal damage 301- A client with chronic kidney disease receives a prescriptions for darbepoetin alfa 40 mcg subcutaneous every 7 days. The darbepoetin alfa vial is labeled, “60 mcg/ml” How many ml should the nurse administer? (Enter numeric value only. If rounding is required, round to the nearest tenth) 0.7 302- A client with metastatic cancer who was taking hydromorphone (Dilaudid) PO at home is now receiving the medication IV while in the hospital. To evaluate if the client is receiving an equianalgesic dose of the Dilaudid, what assessment should the nurse complete? a- Pain scale b- Level of consciousness c- Respiratory rate d- Blood pressure 304- While transferring a client with a chest tube from the bed to a stretcher, the chest becomes disconnected from the water-seal drainage container. The nurse immediately immerses the end of the tube in a container of sterile water. What action should the nurse implement next? a- Begin manual ventilation while returning the client to the bed b- Clamp the chest tube and maintain its distal end in the water c- Prepare a new water-seal system and reattach the chest tube d- Apply pressure to the chest tube site using a petroleum gauze. 305- The nurse is performing a peritoneal dialysis exchange on a client with chronic kidney disease (CKD). Which assessment finding should the nurse report to the healthcare provider? a- The appearance of the returning dialysate fluid is cloudy b- The client complains of a slight shortness of breath during installation c- The amount of the returning dialysate fluid is greater than the amount instilled d- The client complains of abdominal fullness and cramping during instillation 306- A client with type 2 diabetes mellitus is admitted for frequent hyperglycemic episodes and a glycosylated hemoglobin (HbA1c) of 10%. Insulin glargine 10 units subcutaneously once a day at bedtime and a sliding scale with insulin aspart q6h are prescribed. What action should the nurse include in this client‟s plan of care? a- Fingerstick glucose assessment q6h with meals b- Mix bedtime dose of insulin glargine with insulin aspart sliding scale dose c- Review with the client proper foot care and prevention of injury d- Do not contaminate the insulin aspart so that it is available for iv usee- Coordinate carbohydrate controlled meals at consistent times and intervals f- Teach subcutaneous injection technique, site rotation and insulin management 307- The nurse offers diet teaching to a female college student who was diagnosed with irondeficiency anemia following her voluntary adoption of a lacto-vegetarian diet. What nutrients should the nurse suggest this client eat to best meet her nutritional needs while allowing her to adhere to a lacto-vegetarian diet? a- Combine several legumes and grains such as beans and rice form complete proteins b- Drink whole milk instead of skim milk to enhance the body´s production of amino acids c- Increase amount of dark yellow vegetables such carrots to fortify iron stores d- Take vitamin K 10mg PO daily to daily to enhance production of red blood cells 308- What information should the nurse include in the discharge teaching plan of a client with low back pain who is taking cyclobenzaprine to control muscle spasms? a- Take this medications on an empty stomach b- Avoid using heat or ice to injured muscles while taking this medication c- Use cold and allergy medications only as directed by a healthcare d- Discontinue all nonsteroidal anti-inflammatory medications 309- A client with chronic kidney disease has an arteriovenous (AV) fistula in the left forearm. Which observation by the nurse indicates that the fistula is patent? a- Distended, tortuous veins in the left hand b- The left radial pulse is 2+ bounding c- Auscultation of a thrill on the left forearm d- Assessment of a bruit on the left forearm 310- A lethargic one-hour-old infant is brought to the nursery for further assessment. In what order should the nurse assess this infant? ( Arrange the first on the top, and the last item on the bottom) Heel stick Respirations Heart rate Temperature 311- When washing soiled hands the nurse first wets the hands and applies soap. The nurse should complete additional actions in which sequence? (Arrange from first action on top to last action on botton.) RESPONDIDA Rub hands palm to palm Interlace the fingersDry hands with paper towel turn off water faucet 312- The nurse changes a Pleuravac® chest drainage system for a client whose chest collection unit is filled to capacity. After unclamping the attached chest tube, the nurse turns on the suction to 20 cm water pressure, and 125 ml of serosanguineous drainage is obtained. Which location on the Pleuravac® indicates this volume of drainage from the chest tube? (Click the chosen location. 313- A client with a pneumothorax is admitted for insertion of a chest tube that is attached to suction with a chest drainage system. The nurse verifies that the suction level is at 20 cm water pressure. Which location on the chest drainage system indicates this finding? 314. An older male client is admitted with the medical diagnosis of possible cerebral vascular accident (CVA). He has facial paralysis and cannot move his left side. When entering the room, the nurse finds the client‟s wife tearful and trying unsuccessfully to give him a drink of water. What action should the nurse take? a- Give the wife a straw to help facilitate the client‟s drinking. b- Assist the wife and carefully give the client small sips of water c- Obtain a thickening powder before providing any more fluids. d- Ask the wife to stop and assess the client’s swallowing reflex. 315- A young adult male who was in a motorcycle collision experienced a laceration of the gastrocnemius muscle. Which instruction should the nurse to provide to the practical nurse (PN) who is caring for this client?a- Perform range of motion on the affected limb b- Avoid plantar flexion of the affected limb c- Elevate limb above the heart when lying in bed d- Avoid washing the limb when assisting with bathing 316- A male client receives a thrombolytic medication following a myocardial infarction. When the client has a bowel movement, what action should the nurse implement? a- Obtain a specimen for culture and sensitivity analysis b- Assess for fatty yellow streaks in the client‟s stool c- Send a stool sample to the lab for a guaiac test d- Observe the stool for a clay-colored appearance 317- In assessing an adult client with a partial rebreather mask, the nurse notes that the oxygen reservoir bag does not deflate completely during inspiration and the client‟s respiratory rate is 14 breaths / minute. What action should the nurse implement? a- Encourage the client to take deep breaths b- Remove the mask to deflate the bag c- Increase the liter flow of oxygen d-Document the assessment data 318- At 0600 while admitting a woman for a schedule repeat cesarean section (C-Section), the client tells the nurse that she drank a cup a coffee at 0400 because she wanted to avoid getting a headache. Which action should the nurse take first? a. Ensure preoperative lab results are available b. Start prescribed IV with lactated Ringer‟s c. Inform the anesthesia care provider d. Contact the client‟s obstetrician. 317- The nurse notes that a client has been receiving hydromorphone (Dilaudid) every six hours for four days. What assessment is most important for the nurse to complete? a. Auscultate the client's bowel sounds b. Observe for edema around the ankles c. Measure the client‟s capillary glucose level d. Count the apical and radial pulses simultaneously 318- After a third hospitalization 6 months ago, a client is admitted to the hospital with ascites and malnutrition. The client is drowsy but responding to verbal stimuli and reports recently spitting up blood. What assessment finding warrants immediate intervention by the nurse? a. Bruises on arms and legs b. Round and tight abdomen c. Pitting edema in lower legs d. Capillary refill of 8 seconds 319. A young adult woman visits the clinic and learns that she is positive for BRCA1 gene mutation and asks the nurse what to expect next. How should the nurse respond? a- Explain that counseling will be provided to give her information about her cancer riskb- Gather additional information about the client‟s family history for all types of cancer. c- Offer assurance that there are a variety of effective treatments for breast cancer. d- Provide information about survival rates for women who have this genetic mutation 321. The nurse plans to collect a 24-hour urine specimen for a creatinine clearance test. Which instruction should the nurse provide to the adult male client? a-Urinate at the specified time, discard this urine, and collect all subsequent urine the next 24 hours. b- Cleanse around the meatus, discard first portion of voiding, and collect the rest in a sterile bottle. c- For the next 24 hours, notify nurse when the bladder is full, and the nurse will collect catheterized specimens. d-Urinate immediately into a urinal, and the lab will collect the specimen every 6 hours, for the next 24 hours. 322. During the administration of albuterol per nebulizer, the client complains of shakiness. The client´s vital signs are heart rate 120 beats/minutes, respirations 20 breaths/minute, blood pressure 140/80. What action should the nurse take? a- Administer anxiolytic b- Obtain 12 lead electrocardiogram c- Stop the albuterol administration and restart in 30 minutes d- Educate client about the side effects of albuterol 323. A nurse who works in the nursery is attending the vaginal delivery of a term infant. What action should the nurse complete prior to leaving the delivery room? a- Obtain the infant‟s vital signs. b- Observe the infant latching onto the breast c- Administer vitamin K injection d- Place the ID bands. 324. One week after an above-the-knee-amputation (AKA) of the left leg, a male client seems upset and reports that his left foot feels “numb”. What action should the nurse implement? a- Assess wound for signs of inflammation or drainage. b- Offer assurance that the numb feeling is temporary. c- Assess right foot for signs of diminished circulation. d- Reinforce learning about the cause of this sensation. 325. The nurse is preparing to administer 1.6 ml of medication IM to a 4 month old infant. Which action should the nurse include? a) Select a 22 gauge 1 ½ inch (3.8 cm) needle for the intramuscular injection b) Administer into the deltoid muscle while the parent holds the infant securely c) Divide the medication into two injection with volumes under 1ml d) Use a quick dart-like motion to inject into the dorsogluteal site. Rationale: IM injection for children under 3 of age should not exceed 1ml. divide the dose into smaller volumes for injection in two different sites. 326. A mother brings her 4-month-old son to the clinic with a quarter taped over his umbilicus and tells the nurse the quarter is supposed to fix her child‟s hernia. Which explanations should the nurse provide? a) This hernia is a normal variation that resolves without treatment. b) Restrictive clothing will be adequate to help the hernia go away. c) An abdominal binder can be worn daily to reduce the protrusion. d) The quarter should be secured with an elastic bandage wrap. Rational: an umbilical hernia is a normal variation in infants that occurs due to an incomplete fusion of the abdominal musculature through the umbilical ring that usually resolves spontaneously as the child learns to walk. Other choices are ineffective and unnecessary. 327. A mother runs into the emergency department with s toddler in her arms and tells the nurse that her child got into some cleaning products. The child smells of chemicals on hands, face, and on the front of the child's clothes. After ensuring the airway is patent, what action should the nurse implement first? a) Call poison control emergency number. e) Determine type of chemical exposure. f) Obtain equipment for gastric lavage. g) Assess child for altered sensorium. Rational: once the type of chemical is determined, poison control should be called even if the chemical is unknown. If lavage is recommended by poison control, intubation and nasogastric tube may be needed as directed by poison control. Altered sensorium, such as lethargy, may occur if hydrocarbons are ingested 328. A client with arthritis has been receiving treatment with naproxen and now reports ongoing stomach pain, increasing weakness, and fatigue. Which laboratory test should the nurse monitor? h) Sed rate (ESR) i) Hemoglobin j) Calcium k) Osmolality. l) Rational: naproxen can cause gastric bleeding, so the nurse should monitor the client’s hemoglobin to assess for possible bleeding. Other options are not likely to be affected by the used of naproxen and are not related to the client’s current symptoms. 329. The nurse assesses a child in 90-90 traction. Where should did nurse assess for signs of compartment syndrome?330. A native-American male client diagnosed with pneumonia, states that in addition to his prescribed medical treatment of IV antibiotics he wishes to have a spiritual cleaning performed. Which outcome statement indicates that the best plan of care was followed? a) Identifies his ethnocentric values and behaviors b) States an understanding of the medical treatment c) Participated actively in all treatments regimens d) Expresses a desire for cultural assimilation Rationale: indicates active participation by the client, which is required for treatment to be successful. The best plan of care should incorporate the valued and treatments of both cultures and in this case there is no apparent cultural clash between the two forms of treatment. The client has already identify he's cultural values (A). (B) Only considers one of the two treatment modalities desired by the client the client has already chosen how he wishes to assimilate his cultural values with the prescribed medical treatment (D). 331. When assessing a male client, the nurse notes that he has unequal lung expansion. What conclusion regarding this finding is most likely to be accurate? The client has A collapsed lung A history of COPD A chronic lung infection Normally functioning lungs 332. A client has both primary IV infusion and a secondary infusion of medication. An infusion pump is not available. The nurse needs to determine the current rate of infusion of the primary IV. Where should the nurse observe to determine the rate of infusion? 333. During a visit to the Planned Parenthood clinic, a young woman tells the nurse that she is going to discontinue taking the oral contraceptives she has taken for three years because she wants to get pregnant. History indicates that her grandfather has adult onset diabetes and that she was treated for Chlamydia six months ago, which factor in this client‟s history poses the greatest risk for this woman‟s pregnancy? A. Family history of adult onset diabetes. B. Treatment for chlamydia in the past year C. Client‟s age and previous sexual behavior D. Three year history of taking oral contraceptives334. A client is being treated for syndrome of inappropriate ant diuretic hormone (SIADH). On examination, the client has a weight gain of 4.4 lbs (2 kg) in 24 hours and an elevated blood pressure. Which intervention should the nurse implement first? A. Ensure client takes a diuretic q AM B. Obtain serum creatinine levels daily C. Measure ankle circumference D. Monitor daily sodium intake 335. Oxygen at 5l/min per nasal cannula is being administered to a 10-year-old child with pneumonia. When planning care for this child, what principle of oxygen administration should the nurse consider? A. Taking a sedative at bedtime slows respiratory rate, which decreases oxygen? B. Avoid administration of oxygen at high levels for extended periods. C. Increase oxygen rate during sleep to compensate for slower respiratory rate. D. Oxygen is less toxic when it is humidified with a hydration source. 336. A male client with diabetes mellitus type 2, who is taking pioglitazone PO daily, reports to the nurse the recent onset of nausea, accompanied by dark-colored urine, and a yellowish cast to his skin. What instructions should the nurse provide? A. “You have become dehydrated from the nausea. You will need to rest and increase fluid intake” B. “You need to seek immediate medical assistance to evaluate the cause of these symptoms” C. A urine specimen will be needed to determine what kind of infection you have developed” D. use insulin per sliding scale until the nausea resolves, and then resume your oral medication” 337. In assessing a client twelve hour following transurethral resection of the prostate (TURP), the nurse observes that the urinary drainage tubing contains a large amount of clear pale pink urine and the continuous bladder irrigation is infusing slowly. What action should the nurse implement? a. Ensure that no dependent loops are present in the tubing. b. Increase the rate of continues bladder irrigation c. Manually irrigate the catheter with sterile normal saline d. Clamp the caterer above the draining tubing 338. Which problem reported by a client taking lovastatin requires the most immediate fallow up by the nurse? a. Diarrhea and flatulence b. Abdominal cramps c. Muscle pain d. Altered taste Rationale: statins can cause rhabdomyolysis, a potentially fatal disease of skeletal muscle characterized by myoglobinuria and manifested with muscle pain, so this symptom should immediately be reported to the HCP. 339- An older client is referred to a rehabilitation facility following a cerebrovascular accident (CVA) The client is aphasic with left-side paresis and having difficulty swallowing. Which intervention is most important for the include in the client pan of careA. Initiate passive range of motion exercises b. Facilitate a consultation for speech therapy c. Use pictures and gestures to communicate d Arrange for daily home care assistance 340-A client who is admitted with palpitations. Chest discomfort and shortness of breath is anxious while reporting a history of a mitral stenosis related to rheumatic fever as a child. The cardiac monitor displays a supraventricular tachycardia (SVT) at a rate of 180 beats/minute. Vital signs include heart rate 156 beats/minute, respiration 22 breaths /minute, and blood pressure 100/84. Which intervention should the nurse implements first? a. Obtain emergency cart and give midazolam (versed) IV per protocol b. Prepare bedside cardiac defibrillator with synchronized cardio version c. Explain the importance of immediate synchronized cardioversion d. Call 12 lead electrocardiogram and cardiac isoenzymes. 341. To prevent infection by auto contamination during the acute phase of recovery from multiple burns, which intervention is most important for the nurse to implement? a. Implemented protective isolation b. Avoid sharing equipment between multiple clients c. Use gown, mask, and glove with dressing change d. Dress each wound separately 342. When administering ceftriaxone sodium (Rocephin) intravenously to a client before…most immediate intervention by the nurse? m) Stridor n) Nausea o) Headache p) Pruritis 343. The nurse is interacting with a female client who is diagnosed with postpartum depression. Which finding should the nurse document as an objective sign of depression? (Select all that apply) a. Interacts with a flat affect. b. Avoids eye contact. c. Has a disheveled appearance d. Report feeling sad e. Expresses suicidal thoughts 344- A client with a history of chronic obstructive pulmonary disease (COPD) is admitted with pneumonia. Vital signs include: a. Bilateral diffuse wheezing b. temperature of 100. 5 c. Yellow expectorated sputum d. Shortness of breath345. An adult woman who has a history of interior myocardial infarction, esophageal reflux, and type 1 diabetes mellitus (DM) is admitted to the telemetry unit for sudden onset of dizziness with palpation and a burning sensation in her chest. Witch interventions should the nurse implement first? a. Review client‟s last meals choices b. administers an oral antacid c. Assess blood glucose level d. Evaluate telemetry cardiac rhythm 346- An adult client is exhibit the maniac stage of bipolar disorder is admitted to the psychiatric unit. The client has lost 10 pounds in the last two weeks and has no bathed in a week “I‟m trying to start a new business and “I‟m too busy to eat”. The client is oriented to time, place, person but not situation. Which nursing problem has the greatest priority?  Hygiene-self-care deficit  Imbalance nutrition  Disturbed sleep pattern  Self-neglect Rationale: The client’s nutritional status has the highest priority at this time, and finger foods are often provided, sothe client who is on the maniac phase of bipolar disease can receive adequate nutrition. Other options are nursing problems that should also be addresses with the client’s plan of care, but at this stage in the client’s treatment, adequate nutrition is a priority 347- A male client who was discharged 3 days ago after an exploratory laparoscopic biopsy is admitted to the hospital with a warm, tender, and swollen left. The nurse is preparing to initiate heparin therapy. What additional intervention should the nurse include in this client plan of care? a. Administer the client‟s routine daily aspirin b. Encourage the client to dangle his legs frequently c. Maintain the client on bed rest d. Encourage a diet high in iron and ascorbic acid 348- The nurse is teaching a group of women about osteoporosis and exercises. The nurse should emphasize the need for which type of regular activity. a- Core strengthening b- Muscle stretching and toning c- Weight – bearing exercise d- Aerobic exercise 349- The mother of an adolescent female tells the clinic nurse that after every meal her daughter goes to the bathroom, looks the door, and vomits. Which physical assessment should the nurse implement if bulimia is suspect? a. Current height and weight b. Length of the last menses c. skin of palms of the hand d. Condition of tooth enamel 350- A client who has a herniated intervertebral lumbar disc is experiencing severe pain in the right leg. What pathophysiological process explains this client‟s pain?a. Nerve signal interruption from involved joints b. Stress fracture of the lumbar vertebral bodies c. Pressure of the surrounding lumbar tissue. d. inflammation of the surrounding lumbar tissue 351- The nursing staff on a medical unit includes a registered nurse (RN), practical nurse (PN), and unlicensed assistive personnel (UAP). Which task should the charge nurse assign to the RN? a- Transport a client who is receiving IV fluid to the radiology department b- Administer PRN oral analgesics to a client with a history of chronic pain. c- Supervise a newly hired graduated nurse during an admission assessment d- Complete ongoing focused assessments of a client with wrist restraints 352. A neonate whose mother used cocaine during pregnancy is demonstrating extreme irritability, a shrill cry and frequent vomiting. What action should the nurse perform first? a. Burp the infant eliminate gas b. Obtain a serum drug screen c. wraps the infant in warm blankets d. request a neurology assessment 353. An unresponsive male victim of a diving accident is brought to the emergency department where it is determined that immediate surgery is required to save his life. The client is accompanied by a close friend, but no family members are available. What action should the nurse take first. a. Ask he man‟s friend to sign the informed consent since the client is unresponsive b. Notify the unit manager that an emergency court order is needed to allow surgery. c. Continue to provide the support until a thorough search for a guardian is completed. d. Carry on with surgical preparation of the client without a signed informed consent 354- When assessing a multigravida, the first postpartum day, the nurse finds a moderate amount of lochia rubra, with the uterus firm, and three fingerbreadths above the umbilicus. What action should the nurse implement first?  Massage the uterus to decrease atony  Check for a destined bladder  Increase intravenous infusion  Review the hemoglobin to determined hemorrhager Rationale: a fundus that is dextroverted (up to the right) and elevated above the umbilicus is indicative of bladder distension/urine retention. 355. A female client with fibromyalgia asks the nurse to arrange for hospice care to help her manage the severe, chronic pain. Which interdisciplinary team member should the nurse consult to assist the client? a- Contact a hospice nurse for an evaluation b- Arrange an appointment with a pain specialist c- Ask for a consultation with a psychologist d- Form an interdisciplinary team for evaluation356. A client with hemorrhoids ask for information about a high fiber diet. Which breakfast menu items should the nurse suggest? (select all that a apply) a. Cup of raspberries b. Scrambled eggs c. Raising bran muffins d. Bowl of oatmeal e. Bacon slices 357. The nurse is monitoring a client who is newly diagnosed with adrenal insufficiency. The client expresses chronic fatigue and requires assistance to the bathroom. The client is anxious about having a chronic disease that could affect the ability to work. Which intervention should the nurse implement Firs.? a. Place a portable toilet a bedside b. Initiate fall prevention measures c. provides uninterrupted time for rest d. Administer an antianxiety medication 358- A client with a large pleural effusion undergoes a thoracentesis. Following the procedure, which assessment finding warrants immediate intervention by the nurse?  The client has asymmetrical chest wall expansion  The clients complain of pain at the insertion site  The client chest‟s x-ray indicates decreased pleural effusion  The client‟s arterial blood gases are pH 7.35, PaO2 85, Pa CO2 35, HCO3 26. 359- An unlicensed assistive personnel (UAP) leaves the unit without notifying the staff. In what order should the unit manager implement these interventions to address the UAP‟s behavior? (Place the actions in order from first on top to last on bottom). A- Note date and time of the behavior. B- Discuss the issue privately with the UAP. C- Plan for scheduled break times. D- Evaluate the UAP for signs of improvement. 364.. The nurse is planning preoperative teaching plan of a 12-years old child who is scheduled for surgery. To help reduce the child anxiety, which action is the best for the nurse to implement? a. Give the child syringes or hospital mask to play it at home prior to hospitalization. b. Include the child in pay therapy with children who are hospitalized for similar surgery. c. Provide a family tour of the preoperative unit one week before the surgery is scheduled. d. Provide doll an equipment to re-enact feeling associated with painful procedures. Rationale: School age children gain satisfaction from exploring and manipulating their environment, thinking about objectives, situations and events, and making judgments based on what they reason. A tour of the unit allows the child to see the hospital environment and reinforce explanation and conceptual thinking. 360- The charge nurse in a critical care unit is reviewing clients‟ conditions to determine who is stable enough to be transferred. Which client status report indicates readiness for transfer from the critical care unit to a medical unit? a. Chronic liver failure with a hemoglobin of 10.1 and slight bilirubin elevation b. Pulmonary embolus with an intravenous heparin infusion and new onset hematuria. c. Adult respiratory distress syndrome with pulse oximetry of 88% saturation. d. Myocardial infarction with sinus bradycardia and multiple ectopic beats. 366-. A 12-year-old client who had an appendectomy two days ago is receiving 0.9% normal saline at 50 ml/hour. The client's urine specific gravity is 1.035. What action should the nurse implement?  Evaluate postural blood pressure measurements  Obtain specimen for uranalysis  Encourage popsicles and fluids of choice  Assess bowel sounds in all quadrants Rationale: specific gravity of urine is a measurement of hydration status (normal range of 1.010 to 1.025) which is indicative of fluid volume deficit when Sp Gr increases as urine becomes more concentrated. 367- An older male client with type 2 diabetes reports that he experiences leg pain when walking short distances, and that the pain is relieved by the rest. Which client behavior indicates an understanding of healthcare teaching to promote more effective arterial circulation? A. Inspects feet daily for any irritation or skin breakdown B. Consistently applies TED hose before getting dressed in the morning C. Completely stops cigarette/ cigar smoking D. Frequently elevates legs throughout the day 368- The nurse is assessing the skin of a client who is at risk for becoming infected with the human immunodeficiency virus (HIV). Which finding requires immediate intervention? A. Patches of dry, flaky skin B. Numerous moles on the chest and back C. Ecchymoses on the legs D. Purplish- red raised lesions 369- Which conditions are most likely to respond to treatment with antihistamines? (select all that apply.) A. Otitis media B. Myocarditis C. Bronchitis D. Contact dermatitis E. Allergic rhinitis 370. The nurse is assigning care of a client with prostatitis to a practical nurse (PN). What instruction should the nurse provide the PN regarding care of this client? A. Maintain contact isolation B. Restrict oral fluid intakeC. Strain all urine D. Avoid urinary catheterization 371. A male client with a history of heavy alcohol intake is admitted with acute pancreatitis. The client reports severe abdominal pain, radiating to the back. In positioning the client, which instruction should the nurse provide the unlicensed assistive personnel (UAP)? A. Tell client to deep breath and cough every 2 hours. B. Assist client to his side with his knees bent to his chest C. Maintain the client is a supine position D. Mobilize the client to stimulate peristalsis 372. The healthcare provider prescribed furosemide for a 4-year-old child who has a ventricular septal defect. Which outcome indicates to the nurse that this pharmacological intervention was effective? a- Urine specific gravity change from 1.021 to 1.031 b- Daily weight decrease of 2 pounds (0.9Kg) c- Blood urea nitrogen (BUN) increase from 8 to 12 mg/dl (2.9 to 4.3) d- Urinary output decrease of 5 ml/hour 373. A client is receiving continuous ambulatory peritoneal dialysis since the arteriovenous (AV) graft in the right arm is no longer available to use for hemodialysis. The client has lost weight, has increasing peripheral edema, and has a serum albumin level of 1.5 g/ dl (15g/L). Which intervention is the priority for the nurse to implement? a- Evaluate patency of the AV graft for resumption of hemodialysis b- Instruct the client to continue to follow the prescribed fluid restrictions amount c- Recommend the use of support stockings to enhance venous return d- Ensure the client receives frequent small meals containing complete proteins. (OK) 374. The nurse is assessing the feet of a client with diabetes mellitus type 1. Which finding requires the most immediate intervention by the nurse? A. Painful corns and calluses over hammer toes on both feet B. Hard, painless nodule over metatarsophalangeal joint of first toe C. Decreased response to pain discrimination on dorsal surface of foot D. Erythema and edema at the base of the left great toe 375- A client develops urticaria on the trunk and neck shortly after a secondary infusion of piperacillin initiated. In what order the nurse implement these interventions? (Arrange the action in order of priority. Highest priority first and least priority las tor at the bottom) a) Stop the infusion b) Assess vital signs c) Contact the health care provider d) Document reaction to the drug e) Initiate and adverse event report 376- The healthcare provider prescribes amoxicillin 1.5. grams PO daily, in equally divided doses to be administered every 8 hours. The medication is available in a bottle labeled. “Amoxicillin suspension 200 mg/ 5 ml.” How many ml should the nurse administer every 8 hours? (Enter numeric value only. If rounding is required, round to the nearest tenth.) 37.5 377- A male client who had a small bowel resection acquired methicillin- resistant Staphylococcus aureus (MRSA) while hospitalized. He was treated and released but is readmitted today because of diarrhea and dehydration. It is most important for the nurse to implement which intervention? Maintain contact transmission precautions 379. In assessing a client at 34-weeks‟ gestation, the nurse notes that she has a slightly elevated total T4 with a slightly enlarged thyroid, a hematocrit of 28%, a heart rate of 92 beats per minute, and a systolic murmur. Which finding requires follow-up? a. Elevated thyroid hormone level. b. Hematocrit of 28%. c. Heart rate of 92 beats per minute. d. Systolic murmur. Rational: although physiologic anemia is expected in pregnancy, a hematocrit of 28% is below pregnant norms and could signify iron-deficiency anemia. Other options are normal finding pregnancy 380. A client who will be going to surgery states no known allergies to any medications. What is the most important nursing action for the nurse to implement next? a- Assess client’s knowledge of an allergic response b- Record “no known drug allergies” on preoperative checklist c- Flag “no known drug allergies” on the front of the chart. d- Assess client‟s allergies to non-drug substances. 381. The school nurse is screening students for scoliosis and notes that one student has lordosis. Which finding should the nurse document in the student‟s screening record? a- Posterior curvature that is convex in the thoracic area b- Lateral curvature that creates asymmetry of the shoulders c- Rounded spine from head to hips without concave curves d- Excessive concave curvature of the lumbar spine. 382. A client‟s subjective data includes dysuria, urgency, and urinary frequency. What action should the nurse implement next? a. Collect a clean-catch specimen b. palpates the suprapubic region c. inquires about recent sexual activities d. Instruct to wipe from front to back 383 The nurse is explaining the need to reduce salt intake to a client with primary hypertension. What explanation should the nurse provide?  High salt can damage the lining of the blood vessels  Too much salt can cause the kidneys to retain fluid  Excessive salt can cause blood vessels to constrict  Salt can cause information inside the blood vesselsRationale: Excessive salt intake can contribute to primary hypertension by causing renal salt retention which influence water retention that expands blood volume and pressure (ACD) are not believed to contribute to primary hypertension 384. An adolescent client with non-union of a comminuted fracture of the tibia is admitted with osteomyelitis. The healthcare provider collects bone aspirate specimens for culture and sensitivity and applies a cast to the adolescent‟s lower leg. What action should the nurse implement next?  Administer antiemetic agents  Bivalve the cast for distal compromise  Provide high- calorie, high-protein diet  Begin parenteral antibiotic therapy Rationale: The standard of treatment for osteomyelitis is antibiotic therapy and immobilization. After bond and blood aspirate specimens are obtained for culture and sensitivity, the nurse should initiate parenteral antibiotics as prescribed. 387. The nurse planning group therapy for client in a substance abuse program. The focus of the group is “Risk factor hepatitis” What intervention should the nurse plan for the group? a. Include only clients who have hepatitis b. Summarize what the group talked about c. limit the group to no more than seven clients d. talk to client individually before the group 388. The nurse is planning to administer two medications to a client at 0900. Which property of the drugs, if shared by both drug, indicates a need to closely monitor the client for toxicity? a. Low bioavailability b. short half life c. high therapeutic index d. highly protein bound 389. When developing a teaching plan for a client newly diagnosed type 1 diabetes, the nurse should explain that an increased thirst is an early sign of diabetic ketoacidosis (DKA) which action should the nurse instruct the client to implement if this sign of DKA occurs? a. Resume normal physical activity b. Give a dose of regular insulin as prescribed c. Drink electrolyte replacement d. Measure urine output over 24 hours 390. A client with cirrhosis of the liver is having numerous, liquid, incontinent stool, and continues to be confused. In reviewing the client‟s laboratory studies, the nurse identifies an elevated serum ammonia level. Based in this finding. Which prescription is most important for this client to receive? a. IV human albumin b. Lactulose c. Furosemide d. Loperamide391. A client with asthma is manifesting inspiratory and expiratory wheezes and decreased forced expiratory volume. Which prescribed drug class should the nurse administer first to the client? a. Leukotriene modifiers b. Inhaled corticosteroids c. Anticholinergic d. Inhaled short acting beta 2 agonist . 392. The nurse is taking the blood pressure measurement of a client with Parkinson disease. Which information in the client‟s admission assessment is relevant to the nurse plan to taking the blood pressure reading (select all that apply) a. frequent drooling b. occasional nocturia c. frequent syncope d. flat affect e. blurred vision 393. In formulating the nursing care plan for a client diagnosed with Parkinson‟s disease, which nursing problem has the highest priority? a. Risk for constipation relative to immobility. b. Risk for aspiration relative to muscle weakness c. Self-care deficit relative to motor disturbance d. Impaired physical mobility relative to muscle rigidity 394. A female adolescent client admitted to the hospital because she wrote a suicide note to her teacher at school. On the second day of hospitalization, the with the treatment team. After the team meeting, the client leaves in tears and hoes to her room. Which nursing intervention is best? a. Explore the client‟s goals and desire for treatment b. Let the client rest quietly in her room for while c. Ask the treatment team about the client‟s behavior d. Go to the client’s room and ask what happened 395. The nurse takes the morning vital signs of an older resident at a long-term care facility. Vital signs are: temperature 98.7 F (37 C) pulse 84beats/minute, BP 86/64. Which question is most important for the nurse to include in the assessment a. Have you been feeling dizzy? b. Have you been excessively thirsty lately? c. how did you sleep last night? d. Are you having any trouble urinating? 396. When reviewing the discharge plan with a client who has difficulty hearing. The client asks the nurse to repeat the information several times. What action is most important for the nurse to take to ensure the client is receiving and understanding the teaching? a. Maintain a quiet environment while teaching b. Stand in front of client to facilitate lip reading c. provides the instructions to a family member of caretaker d. use a visual aids to reinforce the instruction397-A client who is hospitalized and recently is now confused and lethargic. Which actions should the nurse implement? (Select all that apply) A. Measure capillary glucose level. B. Monitor cardiac telemetry pattern. C. Reduce rate of intravenous fluid infusion. D. Withhold next dose of corticosteroid. E. Initiate fall risk precautions. 398-The nurse is preparing a teaching plan for an older female client diagnosed with osteoporosis. What expected outcome has the highest priority for this client? a- Identifies 2 treatments for constipation due to immobility. b- Names 3 home safety hazards to be resolve immediately. c- State 4 risk factors for the development of osteoporosis. d- Lists 5 calcium-rich foods to be added to her daily diet. 399- To prevent medication errors by an older client who is sometimes confused, which intervention by the by the home a- Have an alert family member administer medication b- Provider education both verbally and in written format c- Instruct the client wear glasses when reading labels d- Encourage taking medications at the same times daily. a) A male client with diabetes mellitus type 2, who is taking pioglitazone PO daily, reports to the nurse the recent onset of nausea, accompanied by dark-colored urine, and a yellowish cast to his skin. What instructions should the nurse provide? A. “You have become dehydrated from the nausea. You will need to rest and increase fluid intake” B. “you need to seek immediate medical assistance to evaluate the cause of these symptoms” C. A urine specimen will be needed to determine what kind of infection you have developed” D. use insulin per sliding scale until the nausea Azithromycin is prescribed for an adolescent female who has lower lobe pneumonia and recurrent chlamydia. What information is most important for the nurse to provide to this client? Use two forms of contraception while taking this drug. c) - blood transfusion= 75 ml d) Herpes Zoster vaccine e)von Willbrandrand‟s desease =riesgo de sangramiento por deficit del factor VIII coagulacion guarding against any injure that might result in bleeding f)-gastric bypass= eating small portion and void high fat food g)-Mag sulfate preeclampsia= continue with the client‟s plan of care h)- Paciente con Metformin y SIDA=dosage change when concurrently take with dolutegravir An elderly female client with osteoarthritis reports increasing pain and stiffness in her right knee and asks how to reduce these symptoms. In responding to the client, the nurse recognizes what pathology as the cause of her symptoms?Destruction of joint cartilage. Amputation phantom limb= cause and nature of this sensations to reinforce preoperative teaching 602- A male client notifies the nurse that he feels short of breath and has chest pressure radiating down his left arm. A STAT 12-lead electrocardiogram (ECG) is obtained and shows ST segment elevation in leads II, II, aVF and V4R. The nurse collects blood samples and gives a normal saline bolus. What action is most important for the nurse to implement? a.Obtain the results for STAT serum cardiac biomarkers b. Asses for contraindications for thrombolytic therapy c.Measure ST-segment height and waveform changes. d. Transfer for percutaneous coronary intervention (PCI) Rationale: ST segment elevation myocardial infarction (STEMI) usually occurs with complete occlusion of an epicardial coronary artery which requires early reperfusion therapy. Screening the client for fibrinolytic therapy (B) is most important to determine PCI option for rapid reperfusion. If the client is not a candidate for fibrinolytic therapy, then transfer to a PCI unit or facility is indicated. Reperfusion therapy should be delayed in STEMI (A). (C) is of significant concern in ECG interpretation with ST-segment depression, not STEMI 603- A client with Addison‟s crisis is admitted for treatment with adrenal cortical supplementation. Based on the client‟s admitting diagnosis, which findings require immediate action by the nurse? (Select all that apply) a- Headache and tremors b- Irregular heart rate c- Skin hyperpigmentation d- Postural hypotension e- Pallor and diaphoresis 604- A client with rapid respirations and audible rhonchi is admitted to the intensive care unit because of a pulmonary embolism (PE). Low-flow oxygen by nasal cannula and weight based heparin protocol is initiated. Which intervention is most important for the nurse to include in this client‟s plan of care? a. Monitor deep vein blood flow using Doppler b. Evaluate daily blood clotting factors. c. Apply antiembolism stockings. d. Maintain strict bed rest. Rationale: Monitoring clotting factors is the most important intervention to include in this client‟s plan of care following oxygen administration, IV fluids and heparin administration to prevent clot enlargement. Ac and D should be included in the client‟s plan of care, but these interventions do not have the priority of B 605- The nurse enters a client‟s room to administer scheduled daily medications and observes the client leaning forward and using pursed lip breathing. Which action is most important for the nurse to implement first? a- Administer schedule medications b- Offer the client PRN anxiolytic c- Assess the lungs for wheezing d- Evaluate the oxygen saturation. Rationale: The client is exhibiting symptoms of an acute exacerbation of a chronic obstructive lung disease such as emphysema. The client… baseline oxygen level should be compared to the current level to determine if respiratory decompensation is occurring. Schedule medications can be administered after completing the oxygen saturation assessment. Respiratory distress often makes a client anxious, which may worsen the symptoms, so should be considered after implementing D. Assessing the lung for wheezing does not reveal further respiratory compromise 606- During a clinic visit, a client with a kidney transplant ask, “What will happen if chronic rejection develops?” which response is best for the nurse to provide? a- The immunosuppressant medication will be increased until the rejection subside b- Dialysis may be necessary until the chronic rejection can be reversed. c- Dialysis would need to be resumed if chronic rejection becomes a reality d- A different combination of immunosuppressant medications will be implemented. Rationale: Chronic rejection is managed conservatively by treating the symptoms until dialysis is needed. Immunosuppressant medication dosage are not increased when chronic rejection occurs, but are during acute rejection. 607- The nurse enters a client‟s room and observe the unlicensed assistive personnel (UAP) making an occupied bed as seen in the picture. What action should the nurse take first? a- Provide the gloves for the UAP to apply b- Offer to help reposition the client c- Instruct the UAP to raise the bed level d- Place the side rails in an up position Rationale: To maintain the client safety, it is most important for the nurse to place the side rails in a up position to reduce the risk of falls and injury. A, B and C can then be completed. 608- A client is receiving continuous bladder irrigation via a triple-lumen suprapubic catheter that was placed during prostatectomy. Which report by the unlicensed assistive personnel (UAP) requires intervention by the nurse? a- Pale pink urine output b- Dark red clot in urine c- Leakage around catheter insertion site d- Urinary output greater than 90 ml/hour. Rationale: After genitourinary surgery, the client is at risk for blood clots and mucus fragments occluding the catheter. Leakage of urine around the suprapubic insertion site indicates blockage of the catheter that causes urine back-up resulting in bladder distention and overflow leakage around the catheter. Pink urine and clots are normal finding 609- A client with bleeding esophageal varices receives vasopressin (Pitressin) IV. What should the nurse monitor for during the IV infusion of this medication? a- Chest pain and dysrhythmia b- Vasodilation of the extremities c- Hypotension and tachycardia d- Decreasing GI cramping and nausea. Rationale: In large doses, vasopressin may produce increased blood pressure, coronary insufficiency, myocardial ischemia or infarction and dysrhythmias. 610- A male client with cancer who has lost 10 pounds during the last months tells the nurse that beef, chicken, and eggs, which used to be his favorite foods, now they taste “bitter”. He complains that he simply has no appetite. What action should the nurse implement? a- Instruct the client to add ground beef and chicken in small amount to casseroles. b- Encourage the client to try to eat these foods in moderation despite the taste c- Advise the client to replace the bitter-tasting foods with fruits and vegetables. d- Suggest the use of alternative sources of protein such as dairy products and nuts. Rationale: Beef, chicken, and eggs are good source of protein. To promote weight gain and adequate protein intake, the nurse should teach the client about another source of protein. Attempting to eat food that cause a bitter taste A and B is likely to increase the client‟s anorexia. C does not provide a sufficient source of protein. 611- A nurse plans to call the healthcare provider to report an 0600 serum potassium level of 2 mEq/L or mmol/L (SI), but the charge nurse tells the nurse that the healthcare provider does not like to receive early morning calls and will make rounds later in the morning. What action should the nurse make? a- Contact the healthcare provider immediately to report the laboratory value regardless of the advice. b- Call the lab to draw an additional blood sample for a repeat evaluation of the potassium level STAT. c- Flag the client‟s medical record to alert the healthcare provider immediately upon arrival to the unit. d- Ask the charge nurse to contact the healthcare provider with the laboratory results by midmorning. Rationale: A serum potassium level of 2 mEq/L or mm/L (SI) is dangerous low and requires immediate intervention A to prevent potentially fatal cardiac dysrhythmias, regardless of the charge nurse concern regarding disturbing the healthcare provider, B, C and D may result in a potentially fatal delay in responding to the hypokalemia. 612- Which actions should the nurse implement with auscultating anterior breath sounds? (Place the first action on top and last action on the bottom.) 1. Place stethoscope in suprasternal area to auscultate from bronchial sounds 2. Auscultate bronchovesicular sounds from side to side of the first and second intercostal spaces 3. Displace female breast tissue and apply stethoscope directly on chest wall to hear vesicular sounds 4. Document normal breath sounds and location of adventitious breath sounds Rationale: Begin auscultation of anterior breath sounds over the trachea and larynx to identify bronchial breath sounds. Assessment should proceed down the anterior chest from side to hear bronchovesicular sounds, which are located over major bronchi around the upper sternum in first and second intercostal spaces. To hear vesicular sounds over peripheral lung fields where air flows through smaller airway, the breast tissue should be displaced so the stethoscope lies directly on chest wall. Documentation should include normal breath sounds and any adventitious findings. 613- Methylprednisolone (Solu-Medrol) 100 mg IV is prescribed for a client. The medication comes in a vial labeled “125 mg per ml” How many ml should the nurse administer. (enter numeric value only. If rounding is required, round to the nearest tenth) 0.8 ml 614- An adult woman who has a history of inferior myocardial infarction, esophageal reflux, and type 1 diabetes mellitus (DM) is admitted to the telemetry unit for sudden onset of dizziness with palpitations and burning sensation in her chest. Which intervention should the nurse implement first? a- Review the client‟s last meal choice b- Administer an oral antacid c- Assess blood glucose level d- Evaluate telemetry cardiac rhythm. 615- A client with Huntington‟s disease is exhibiting choreic movements. Which action the nurse implement first? a- Assess the client‟s vital signs and breath sounds b- Place padding around the side rails of the beds c- Instruct the client to perform deep breathing d- Document the behavior in the client‟s medical record. Rationale: The nurse‟s highest priority is to initiate measures to protect the client (B) because choreic movements are involuntary hyperkinetic movements, which may result in injury to the client. This is typical of Huntington‟s disease and does not cause significant changes in A. Because the movements are involuntary C may help the client relax, but will not reduce the movements. D should be addressed after safety issues are resolved.616- A client who is in labor tells the nurse: “I just felt a large gush of water- I think my water broke”. After assessing the FHR of 140 beats per minute, which action should the nurse implement next? a- Perform a sterile vaginal exam b- Turn to the left lateral position c- Monitor maternal vital signs d- Document fetal heart rate. 617- A client‟s cardiac monitor displays new prominent U waves. What action should the nurse implement? Select all that apply. a- Administer a PRN sublingual nitrate tablet b- Assess rhythm for ST depression c- Evaluate client‟s current potassium level d- Notify the rapid response team immediately e- Obtain a 12 lead electrocardiogram 618- A female client experiencing her four episode of heart failure insist that she has followed a strict diet and taken her prescribe medications. What intervention is most important to for the nurse to include in the client‟s discharge teaching plan? a- Describe possible medication side effects b- Schedule heart failure educator follow-up c- Encourage recording intake and output d- Instruct the client to keep a diet journal. 619- A female client with vomiting and diarrhea for the past 2 days arrives in the fast-track clinic complaining of thirst but is… tolerate oral fluid at this time. Her blood pressure is 110/70 with heart rate of 80 beats/minute, and her standing blood pressure is 90/60 with a heart rate of 120 beats/minute. Which action is most important for the nurse to take? a- Contact the healthcare provider about initiating parenteral fluids b- Monitor the client‟s orthostatic vital sign every 2 hours. c- Review serum sodium and blood urea nitrogen (BUN) results d- In 2 hours offer the client small amounts of clear oral liquids. Rationale: A sign of dehydration is thirst, which does usually occur unless there is 25% fluid deficiency. Parenteral fluids should be considered, so the healthcare provider should be contacted. B, C and D should be implemented but addressing the client need for parenteral fluid is most important 620- The nurse observes a UAP begin to removes exam gloves after emptying a bedpan containing feces. The UAP slides two fingers inside one of the gloves and begins to roll the glove off. What action should the nurse implement? a- Remind the UAP to discard the gloves in the biohazard container after removal b- Suggest that the UAP roll both of the gloves and insider out at the same time c- Advise the UAP that the technique being used will result in hand contamination d- Instruct the UAP to use two pairs of gloves when fecal contamination is likely. Rationale: By sliding glove fingers inside the other glove the UAP‟s hand become contaminated. Gloves should be discarded in the trash unless heavily contaminated with body fluids is likely to result in contamination of both hands. Double gloving is not necessary when performing activities as emptying a bedpan. 621- An adult woman who has recently diagnosed with type 2 diabetes mellitus (DM) is seen in the clinic for laboratory test. The client height is 5 feet and 2 inches (152.5 cm) and weight is 165 pounds (74.8kg). Her recent laboratory finding are describe above. IN planning nutrition. Her recent laboratory finding are described above. In planning nutrition teaching for this client, what diet modifications should the nurse recommend? Select all that apply. a- Reduce daily fat intake to 10% of total calories b- Increase dietary fiber such as whole grains c- Decrease processed carbohydrate in diet d- Eliminate alcohol intake except for special occasions. e- Restrict protein to 10% of total calories in diet. 622- An experiences UAP is hired to work with in an antepartal clinic. Which assignments is best for this new employee‟s first day of work at the clinic? a- Accompany the health care provider during pelvic examinations b- Discuss good dietary choices with clients using a char provided by the clinic c- Take initial vital signs and urine checks for glucose along with the nurse. d- Schedule clients for their next antepartal visit to the clinic. Rationale: The nurse would have an opportunity to assess the UAP‟s skills such as vital signs, glucose monitoring and communication by working with the UAP. A new employee would not be familiar with the location of supplies or the routines of the healthcare providers. B is responsibility of the RN or aa PN if standard plan is used. Evaluation of client‟s condition and knowledge of clinic protocol are required when scheduling a client „s next antepartal visit, which would require judgment of the RN. 623- The home health nurse assesses a male client who is on the waiting list for a liver transplant. The client expresses fear that he will die before a liver becomes available. How should the nurse respond? a- Encourage the client to discuss how is he is copying with his fearfulness b- Offer to notify the healthcare provider about his feeling of fearfulness c- Explain that he will be moved up on the list if his condition worsens.d- Assess the client‟s current mental status and over-all cognitive functions. 624- The healthcare provider prescribes a low dose heparin protocol at 18 units/kg/hr. for a client with a possible pulmonary embolism. This client weighs 144 pound. The available solution is labeled, Heparin Sodium 25,000 Units in 5% dextrose… 250ml. The nurse should program the pump to deliver how many m/hr. (enter numeric value only. If round to the nearest whole number) 12 Rationale: Client‟s weight 144 lbs./2.2 = 65 kgs 65 kg x 18 units/kg/hr.= 1170 units/hour. 25,000 units of heparin divided by 250ml of D5W = 100 units heparin per 1 ml of solution. Using the formula D/H x A=X 625- The nurse is planning care for a client at 30-weeks gestation who is experiencing…. Most important in preventing this fetus from developing respiratory distress syndrome… a- Butorphanol 1mg IV push q2h PRN pain b- Betamethasone (celestone) 12 mg deep IM c- Terbutaline (Brethine) 0.25 mg subcutaneously q15minutesx 3 d- Ampicillin 1 Gram IV push q8h. 626- The nurse places one hand above the symphysis while massaging the fundus of a multiparous… 15 mints after delivering a 7 pound and 10 ounces‟ infant. Which information should the nurse pr….. a- Clots may form inside a boggy uterus and need to be expelled b- The uterus should to be firm to prevent an intrauterine infection c- A firm uterus prevents the endometrial lining from being sloughed d- Both the lower uterine segment and the fundus must be massaged. Rationale: A boggy uterus indicates the uterus is not contracting adequately, the uterus is not contacting adequately, which means that the uterus…. Intrauterine bleeding that causes the uterus become a reservoir to hold blood clots [Show More]

Last updated: 1 year ago

Preview 1 out of 76 pages

Reviews( 0 )

$10.00

Add to cart

Instant download

Can't find what you want? Try our AI powered Search

OR

GET ASSIGNMENT HELP
94
0

Document information


Connected school, study & course


About the document


Uploaded On

Jun 16, 2021

Number of pages

76

Written in

Seller


seller-icon
QuizMaster

Member since 3 years

11 Documents Sold


Additional information

This document has been written for:

Uploaded

Jun 16, 2021

Downloads

 0

Views

 94

Document Keyword Tags

Recommended For You


$10.00
What is Browsegrades

In Browsegrades, a student can earn by offering help to other student. Students can help other students with materials by upploading their notes and earn money.

We are here to help

We're available through e-mail, Twitter, Facebook, and live chat.
 FAQ
 Questions? Leave a message!

Follow us on
 Twitter

Copyright © Browsegrades · High quality services·